Что добавляют воду в кислоту или кислоту в воду: «Почему нельзя приливать воду в серную кислоту для её разбавления?» – Яндекс.Кью

Что добавляют воду в кислоту или кислоту в воду: «Почему нельзя приливать воду в серную кислоту для её разбавления?» – Яндекс.Кью

Содержание

Что льют кислоту в воду или наоборот. Неправильное смешивание концентрированной серной кислоты с водой (как это выглядит на практике)

Как смешать два жидких вещества? Например, какую-нибудь кислоту и воду? Казалось бы эта задача из серии «дважды два – четыре». Что может быть проще: слить две жидкости вместе, в какой-нибудь подходящей емкости, и все дела! Или влить одну жидкость в емкость, где уже находится другая. Увы, это та самая простота, которая, по меткому народному выражению, хуже воровства. Поскольку дело может закончиться крайне печально!

Инструкция

Имеется две емкости, в одной из них содержится концентрированная серная кислота, в другой – вода. Как их правильно смешать? Лить кислоту в воду или, наоборот, воду в кислоту? Ценой неверного решения в теории может стать низкая оценка, а на практике — в лучшем случае, сильный ожог.

Почему? А потому, что концентрированная серная кислота, во-первых, гораздо плотнее воды, а во-вторых, чрезвычайно гигроскопична. Иными словами, она активно поглощает воду. В-третьих, это поглощение сопровождается выделением большого количества тепла.

Если в емкость с концентрированной серной кислотой начнут приливать воду, первые же порции воды «растекутся» по поверхности кислоты (поскольку вода гораздо менее плотная), и кислота начнет жадно поглощать ее, выделяя тепло. А этого тепла будет так много, что вода буквально «вскипит» и брызги полетят во все стороны. Естественно, не миновав незадачливого экспериментатора. Обжечься и «чистым» кипятком не очень приятно, а если учесть, что в водяных брызгах наверняка будет еще кислота. Перспектива становится совсем невеселой!

Именно поэтому многие поколения учителей-химиков заставляли своих учеников буквально зазубривать правило: «Сначала вода, потом – кислота! Иначе случится большая беда!» Концентрированную серную кислоту следует добавлять в воду, маленькими порциями, при перемешивании. Вот тогда вышеописанная неприятная ситуация не произойдет.

Резонный вопрос: с серной-то кислотой понятно, а как быть с другими кислотами? Как правильно смешивать их с водой? В каком порядке? Необходимо знать плотность кислоты. Если она плотнее воды, например, концентрированная азотная, ее точно так же, как серную, следует приливать в воду, соблюдая вышеуказанные условия (понемногу, при перемешивании). Ну, а если плотность кислоты очень незначительно отличается от плотности воды, как в случае с уксусной кислотой, тут уж без разницы.

Внимание, только СЕГОДНЯ!

Все интересное

Повышенное внимание и осторожность, а также соблюдение особых мер безопасности — необходимое условие при работе с кислотами. К работе с кислотами допускаются лица, достигшие 18 лет, при этом обязательным условием является прохождение курса…

Сернистая кислота — это неорганическая кислота средней силы. Из-за неустойчивости нельзя приготовить ее водный раствор с концентрацией более 6%, иначе она начнет распадаться на серный ангидрид и воду. Химические свойства сернистой кислотыСернистая…

Серная кислота — маслянистая бесцветная жидкость, не имеющая запаха. Относится к сильным кислотам и растворяется в воде в любых соотношениях. Имеет колоссальное применение в промышленности. Серная кислота — довольно тяжелая жидкость, ее плотность…

Серная кислота по физическим свойствам – тяжелая маслянистая жидкость. Она не имеет запаха и цвета, гигроскопична, хорошо растворяется в воде. Раствор с содержанием h3SO4 менее 70% обычно называют разбавленной серной кислотой, более 70% –…

Хлористоводородная (соляная, HCl) кислота – это бесцветная, очень едкая и ядовитая жидкость, раствор хлорного водорода в воде. При сильной концентрации (38% от общей массы при температуре 20оС окружающей среды) — «дымится», туман и пары…

Серная кислота имеет химическую формулу h3SO4. Это тяжелая маслянистая жидкость, бесцветная или с желтоватым оттенком, который ей придают примеси ионов металлов, например, железа. Серная кислота очень гигроскопична, легко поглощает водяные пары.…

Серная кислота входит в пятерку самых сильных кислот. Необходимость в нейтрализации этой кислоты возникает, в частности, в случае ее утечки и при возникновении угрозы отравления ей. Инструкция 1Молекула серной кислоты состоит из двух атомов…

С давних времен, объясняя, как смешивать концентрированную серную кислоту с водой, учителя заставляли учеников запоминать правило: «Сначала вода, потом – кислота!» Дело в том, что если поступить наоборот, первые же порции более легкой…

Серная кислота, имеющая химическую формулу h3SO4, представляет собою тяжелую, плотную жидкость маслянистой консистенции. Очень гигроскопична, легко смешивается с водой, при этом обязательно следует лить кислоту в воду, ни в коем случае не наоборот.…

В любом автомобиле есть источник тока, этим источником является аккумулятор. Т. к. аккумулятор – это элемент многоразового использования, то его можно подзаряжать и менять в нем электролит. Раньше, в автомобилях применялись как кислотные, так и…

Сульфаты железа – это неорганические химические вещества, они делятся на разновидности. Существует двухвалентный сульфат железа(2) и трехвалентный сульфат железа(3). Есть много способов получения этих сернокислых солей. Вам понадобитсяЖелезо,…

Что происходит, если кислоту соединить с какой-либо солью? Ответ на этот вопрос зависит от того, какая это кислота и какая соль. Химическая реакция (то есть превращение веществ, сопровождающееся изменением их состава) между кислотой и солью может…

Как смешать два жидких вещества? Например, какую-нибудь кислоту и воду? Казалось бы эта задача из серии «дважды два – четыре». Что может быть проще: слить две жидкости вместе, в какой-нибудь подходящей емкости, и все дела! Или влить одну жидкость в емкость, где уже находится другая. Увы, это та самая простота, которая, по меткому народному выражению, хуже воровства. Поскольку дело может закончиться крайне печально!

Инструкция

Имеется две емкости, в одной из них содержится концентрированная серная кислота, в другой – вода. Как их правильно смешать? Лить кислоту в воду или, наоборот, воду в кислоту? Ценой неверного решения в теории может стать низкая оценка, а на практике — в лучшем случае, сильный ожог.

Почему? А потому, что концентрированная серная кислота, во-первых, гораздо плотнее воды, а во-вторых, чрезвычайно гигроскопична. Иными словами, она активно поглощает воду. В-третьих, это поглощение сопровождается выделением большого количества тепла.

Если в емкость с концентрированной серной кислотой начнут приливать воду, первые же порции воды «растекутся» по поверхности кислоты (поскольку вода гораздо менее плотная), и кислота начнет жадно поглощать ее, выделяя тепло. А этого тепла будет так много, что вода буквально «вскипит» и брызги полетят во все стороны. Естественно, не миновав незадачливого экспериментатора. Обжечься и «чистым» кипятком не очень приятно, а если учесть, что в водяных брызгах наверняка будет еще кислота. Перспектива становится совсем невеселой!

Именно поэтому многие поколения учителей-химиков заставляли своих учеников буквально зазубривать правило: «Сначала вода, потом – кислота! Иначе случится большая беда!» Концентрированную серную кислоту следует добавлять в воду, маленькими порциями, при перемешивании. Вот тогда вышеописанная неприятная ситуация не произойдет.

Резонный вопрос: с серной-то кислотой понятно, а как быть с другими кислотами? Как правильно смешивать их с водой? В каком порядке? Необходимо знать плотность кислоты. Если она плотнее воды, например, концентрированная азотная, ее точно так же, как серную, следует приливать в воду, соблюдая вышеуказанные условия (понемногу, при перемешивании). Ну, а если плотность кислоты очень незначительно отличается от плотности воды, как в случае с уксусной кислотой, тут уж без разницы.

В разделе на вопрос Что произойдет если кислоту налить в воду, и наоборот если воду в кислоту заданный автором Профайл удалён
лучший ответ это Нужно наливать серную кислоту в воду тогда все будет ОК.
А если наоборот… то как минимум разорвет пробирку, а как максимум — тебе будет больно, когда начнет кис-та брызгать и обжигать тебя.
вода легче серной кислоты. Температура плавления равна 10,37 °С при теплоте плавления 10,5 кДж/моль. Это много. — если вливать воду в серную кислоту, то вода закипит в процессе вливания и начнет разбрызгиваться.

Ответ от Дёминов Анатолий
[гуру]
Начинает кипеть и брызгаться…

Ответ от электросварщик
[гуру]
реакция))) но при наливании воды в кислоту лучше близко не стоять и не вдыхать и глазки туды не засовывать…. перами обозжет

Ответ от Невроз
[гуру]
слышали такое выражение кислота ходит в гости…

Ответ от Колосовые
[гуру]
слабый раствор кислоты либо воды… =))

Ответ от Їерчилль
[гуру]
Второе можно производить только в комбинезоне химзащиты.

Ответ от Анатолий Подплетний
[гуру]
В первом случае-кислота тяжелее и сразу уходит на дно остается только помешать и будет элекролит! А во втором случае легкая вода начнет разбрызгивать и повыжигает глаза если нет очков!!

Ответ от ******
[гуру]
То ли будешь зрячая — то ли наоборот! Воду в кислоту — НЕЛЬЗЯ КАТЕГОРИЧЕСКИ! Выброс в виде брызг — глаза — лицо — ожоги -слепота — потеря красоты — инвалид по дурости!!

Ответ от Михаил Бармин
[гуру]
Диссоциация (растворение происходит с выделением большого количества тепла. На до вливать вещество с большей плотностью в вещество с меньше плотностью, что приведет к лучшему перемешиванию и меньшей скорости, иначе ВЫБРОС раствора и ПОРАЖЕНИЕ ГЛАЗ!!

Ответ от Ира Юшинова
[новичек]
Смотря какую кислоту. В случае с азотной и соляной ничего страшного не произойдет, кислоты просто нагреются. Соляная нагреется сильнее. В случае с серной реакция будет идти с большим выделением тепла. В первом случае серная кислота опустится на дно, так у воды меньшая плотность и реакция будет проходить безопасно. Во втором случае реакция будет проходить на поверхности с большим выделением тепла. Похожее происходит, когда на раскаленную сковородку капнуть воды. Но в конце концов кислота поглотит воду.

Вы помните это правило безопасности
, рассказанное на школьных уроках химии?

Я тут подумал намедни, а ЧТО будет с Германией и её «швулями»
при ВЛИВАНИИ в эту страну концентрированного потока исламистов
?

Как вы думаете, получится раствор
или взрыв
?!

Возможно вы спросите, а кто такие немецкие «швули»
?

Послушайте рассказ этой девушки:

По утверждению самих немцев, количество «швулей»
в Германии уже давно перевалило за миллион человек
!

В 2001 году, 14 лет назад, немецкое правительство и церковь узаконили гомосексуальные браки
. Как результат, в Германии более 400 тысяч только официально зарегистрированных однополых браков
.

Рост мужского гомосексуализма прямо пропорционален росту эмансипации.

Это Германия, XXI век.

Немецкому обществу навязан очередной миф и стереотип. Сегодня немецкое общество — это общество социальной амнезии. Ему свойственны безропотное следование приказам и установкам, индифферентность личности, комформизм на пару с униформизмом и духовное порабощение. А так же резкий перепад духовного климата (сегодня ещё здравствуют люди, помнящие, как гомосексуалистов сажали в КZ). И это покорно-бездумное состояние человека «массы» сделало его лёгкой добычей демагогии и предрассудка.

Декларируя всеобщее равенство, демократия не терпит разнообразия, если речь не идёт о гомосексуалистах, транссексуалах и травеститах. Им позволена «цветущая множественность»
, а остальных гуманно придушат подушкой политкорректности. На гомосексуализм сегодня в Германии стандартное нейрофизиологическое реакция, как и на все приказы, что спускаются сверху.

Приказано любить — полюбим. Семьдесят лет назад приказали не любить — нелюбили.

Гомосексуализм в Германии стал маргинальным феноменом общественной жизни. В атмосфере вседозволенности он превратился в пандемию.

Институт семьи рухнул под совместным натиском гомосексуалистов, феминисток и атеистов. Мораль и традиция выдавлены в маргинез.

Не только в Германии, но и в других странах мира усилились приметы тотальной болезненности.

Человечество все больше отдаляется от той нормы, которую Эрих Фромм называл «здоровой». Приметами такой жизни являются все возрастающая отчужденность между людьми, усиление эгоцентризма, релятивизация и разрушение традиционных морально-этических ценностей, нарастающая хаотичность жизни, гедонизм и меркантилизм. Источник.

Это была одна сторона медали
, а это другая. Она называется «вливание концентрированного потока мусульман в Германию»
:

И если вновь прибывшие мусульмане
вот так обходятся с полицейскими Германии, то я даже представить себе не могу, какая будет у них реакция на немецких «швулей»
!

А ведь и те, и другие в равной степени пригреты и обласканы
правительством новой Германии!

На мой взгляд, такая «забота»
о немецкой нации со стороны канцлера Германии Ангелы Меркель, это на порядок круче замысел, чем чья-то задумка свести в России воедино бойцов ВДВ и питерских геев!

Как думаете, друзья, смогут ли мусульмане вылечить
Гейропу и Германию от содомии
?

АНЕКДОТ в ТЕМУ от Владимира Вольфовича:

Однажды утром Штирлиц зашёл к Мюллеру. Настроение у того было мрачнее тучи.
— Господин группенфюрер, что случилось? Русские уже в пригородах Берлина?
— Штирлиц, бросьте ваши дурацкие шутки! Мне приснился страшный сон — Германия 2015 года!
— И что же, там всё так плохо?
— Не то слово! Представляете, у нас в Германии канцлер — баба, министр иностранных дел — педераст, на заводах Даймлер-Бенц работают турки, Германия оплачивает долги греков и испанцев, вместо факельных шествий — гей-парады, мы платим деньги евреям и выполняем команды негра из Америки!

При смешивании концентрированной серной кислоты и воды выделяется много тепла. Для химика этот факт очень важен, поскольку и в лаборатории, и в промышленности часто приходится готовить разбавленные растворы серной кислоты. Для этого нужно смешивать концентрированную серную кислоту с водой — не всегда, но часто.

Как смешать концентрированную серную кислоту и воду
?

Во всех учебниках и практикумах настоятельно рекомендуют лить серную кислоту в воду

(тонкой струйкой и при хорошем перемешивании) — а не наоборот: нельзя лить воду в концентрированную серную кислоту!

Почему? Серная кислота более тяжелая, чем вода.

Если лить кислоту тонкой струйкой в воду, то кислота опустится на дно. Тепло, которое выделится при смешивании, рассеется — пойдет на нагрев всей массы раствора, поскольку над слоем кислоты, которая опустилась на дно сосуда, расположено большое количество воды.

Тепло рассеется, раствор нагреется — и ничего плохого не произойдет, особенно, если в процессе добавлении кислоты к воде жидкость хорошо перемешивать.

А что будет, если сделать неправильно

, — в концентрированную серную кислоту добавить воду? Когда первые порции воды попадут в серную кислоту, они останутся на поверхности (поскольку вода легче, чем концентрированная серная кислота). Выделится много

тепла, которое пойдет на нагрев маленького количества

воды.

Вода резко вскипит, в результате полетят брызги серной кислоты и образуется едкий аэрозоль. Эффект может быть примерно такой, как при добавлении воды на горячую сковородку с маслом. Брызги серной кислоты могут попасть в глаза, на кожу и одежду. Аэрозоль серной кислоты не только очень неприятен при вдыхании, но и опасен для легких.

Если стекло не термостойкое — сосуд может треснуть.

Чтобы это правило было легче запомнить, придумывают специальные стишки вроде:

«Сначала вода, а потом кислота — иначе случиться большая беда!».

Используют также специальные фразы для запоминания — «мемы», например:

«Чай с лимоном».

Книги — хорошо, но решил заснять, как выглядит результат неправильного смешивания концентрированной серной кислоты и воды на практике.

Разумеется, со всеми мерами предосторожности: начиная от защитных очков, заканчивая использованием небольших количеств веществ.

Провел несколько экспериментов — я пробовал смешивать серную кислоту с водой (и правильно, и неправильно). Во обоих случаях наблюдался лишь сильный разогрев. А закипания, разбрызгивания, и подобного не происходило.

Для примера, опишу один из экспериментов, проведенный в пробирке. Концентрированной серной кислоты взял 20 мл, воды 5 мл. Обе жидкости комнатной температуры.

Начал добавлять к серной кислоте воду. Вода закипела лишь в момент, когда приливал первые порции воды к кислоте. Новые порции воды погасили кипение. Едкий аэрозоль полетел (к этому я был не готов, пришлось на несколько секунд отойти). Попробовал перемешать алюминиевой проволочкой (то, что было под рукой). Эффекта ноль. Измерил температуру термометром. Оказалось 80 градусов по Цельсию. Эксперимент удался едва ли.

Новый эксперимент провел в колбе: чтобы поверхность соприкосновения двух жидкостей была максимальной (это обеспечит более резкое выделение тепла), а толщина слоя воды над серной кислотой — минимальной. Воду добавлял не всю сразу, а небольшими порциями (чтобы тепло пошло на кипение воды, а не на нагрев всей массы воды).

Итак, в коническую колбу налил около 10-15 мл концентрированной серной кислоты. Воды использовал около 10 мл.

Пока готовился к опыту, кислота под палящим солнцем разогрелась до 36-37 градусов (что градусов на 20 выше, чем начальная температура кислоты в прошлом опыте). Вода в пробирке тоже слегка нагрелась, но не так сильно.
Думаю, это сыграло большую роль в успехе опыта.

При добавлении основной порции воды в серную кислоту заметно летели брызги и едкий аэрозоль. К счастью, их сносило ветром, который дул с моей стороны, поэтому я даже ничего не ощутил.

В итоге, температура в пробирке поднялась выше 100 градусов!

Какие можно сделать выводы? Если нарушить правило, что нельзя добавлять воду к концентрированной серной кислоте

, разбрызгивание происходит не всегда, но оно возможно — особенно, когда вода и кислота теплые. Особенно — если добавлять воду медленно, небольшими порциями и в широкой посуде.

При работе с бОльшими количествами воды и кислоты вероятность резкого разогрева и разбрызгивания возрастает (напоминаю: мы взяли всего несколько миллилитров).

Опыт, который демонстрирует, что нельзя добавлять воду в концентрированную серную кислоту

, описан в практикуме авторов Рипан и Четяну .

Приведу цитату:

Если в концентрированную серную кислоту наливать воду, то первые капли воды, попавшие в нее, моментально превращаются в пар и из сосуда вылетают брызги жидкости. Это происходит по той причине, что вода, обладая небольшим удельным весом, не погружается в кислоту, а кислота ввиду малой теплоемкости не поглощает выделившейся теплоты. При вливании горячей воды наблюдается более сильное разбрызгивание серной кислоты.

Опыт
. Смешивание воды с концентрированной Н 2 SO 4
. Стаканчик с концентрированной серной кислотой ставят на дно большого стакана, прикрытого воронкой. Теплую воду вливают при помощи пипетки (рис. 161). При вливании горячей воды внутренние стенки большого стакана и воронки моментально покрываются брызгами жидкости.

Рис. 161

За неимением стеклянной воронки можно воспользоваться картонной, внутрь которой вводят пипетку с водой.

Если в стакан с водой приливать по каплям или тонкой струей концентрированную серную кислоту, то можно заметить, как более тяжелая серная кислота опускается на дно стакана.

При смешивании концентрированной Н 2 SO 4 со льдом можно одновременно наблюдать два явления: гидратацию кислоты, сопровождающуюся выделением тепла, и плавление льда, сопровождающееся поглощением тепла. Поэтому в результате смешивания можно наблюдать либо повышение, либо понижение температуры. Так, при смешивании 1 кг льда с 4 кг кислоты температура повышается почти до 100°, а при смешивании 4 кг льда с 1 кг кислоты температура снижается почти до -20°.

Кислоты

3.
 КИСЛОТЫ

 

Кислотами
называются
сложные
вещества,
состоящие из
атомов
водорода и
кислотных
остатков.

С
точки зрения
теории
электролитической
диссоциации
кислоты это
электролиты, диссоциирующие
в водных
растворах 
на катионы
только водорода
Н+ и анионы
кислотных
остатков.

Если
кислота
одноосновная,
то она диссоциирует
в одну
ступень:

HCI
H+
+ CI

HNO3 H+ + NO3

Если
кислота
многоосновная,
то она диссоциирует
ступенчато:

H3PO4   H+ + H2PO4     
(первая
ступень),

H2PO4  H+ + HPO42  
(вторая ступень),

HPO42  H+ + PO43    
(третья
ступень).

 

Ступенчатой
диссоциацией
многоосновных
кислот
объясняется
образование
кислых солей.

 

Номенклатура
кислот
.

1. Бескислородные
кислоты  

В бескислородных
кислотах
называется
кислотообразующий
элемент и
добавляется
окончание
водородная:

HCl хлороводородная
кислота

H2S
сероводородная
кислота

2.
Кислородосодержащие
кислоты.

Составление
названий
кислородосодержащих
кислот
рассмотрим
на следующих
примерах:

H2SO4
серная
кислота,

H3AsO4
 мышьяковая
кислота.

Называется
кислотообразующий
элемент с суффиксом
н или ов (если
степень
окисления
элементов
максимальная).

Если
степень
окисления
элемента
промежуточная,
то в
названии 
используется
еще и суффикс
ист:

H2SO3
сернистая
кислота,

H3AsO3
  мышьяковистая
кислота.

 

Когда
элемент
образует
много
кислородсодержащих
кислот
(например,
хлор), то, по
мере убывания
степени
окисления
кислотообразующего
элемента, они
имеют
следующие
названия:

HO4
хлорная
кислота;

HO3
хлорноватая
кислота;

HO2
хлористая кислота;

HO
хлорноватистая
кислота.

 

Иногда
в молекулах
кислородосодержащих
кислот
элемент
имеет
одинаковую
степень окисления,
тогда в
названии
используются
приставки,
которые
указывают на
различное
содержание
воды в
кислоте:

мета
— мало воды,

орто —
много воды.

Например:

P2O5 + H2O →
2HO3
метафосфорная
кислота,

P2O5 + 3H2O →
2H3O4
ортофосфорная
кислота.

 

 

При
составлении
формул

придерживаются
следующих
правил:

1. Бескислородные
кислоты.

Зная
название
кислоты,
записываем
сначала
водород, а затем
кислотообразующий
элемент.
Степень окисления
водорода в
кислотах
всегда +1. Степень
окисления
элемента
отрицательная.
Она равна
номеру
группы ПСЭ (в
которой
находится
элемент)
минус восемь.

Например:
сероводородная
кислота
элемент сера,
расположен в
шестой
группе ПСЭ. 6 — 8 = -2.
Степень
окисления серы
2. Записываем
символы
водорода и
серы Н+S-2, т.к.
молекула электронейтральна,
то формула
кислоты
будет Н2S.

2.
Кислородсодержащие
кислоты.

По
суффиксам в
названии
кислоты
определяем
степень
окисления кислотообразующего
элемента. Эта
степень
окисления
сохраняется
в кислотном
оксиде. По
приставке в
названии
определяем
количество
воды в
кислоте.

Например:
метафосфорная
кислота
кислотообразующий
элемент
фосфор.
Суффикс н
показывает,
что он имеет
максимальную
степень
окисления,
фосфор в
пятой группе
ПСЭ,
следовательно,
максимальная
степень
окисления +5,
она
сохраняется
и в оксиде
фосфора Р2+5О5-2.
Приставка
мета
говорит о
том, что воды
в кислоте
минимальное
количество.

В
формулах
кислородосодержащих
кислот
сначала
записывается
водород, затем
кислотообразующий
элемент и
кислород. Индексами
выравнивают
число
положительных
и
отрицательных
зарядов. Если
они четные,
то их
сокращают и
ставят перед
формулой соответствующий
коэффициент.

+5  -2

P2O5 + H2O →
H2P2O6
2HPO3
метафосфорная
кислота,

+5  -2

P2O5 + 2H2O →
H4P2O7
пирофосфорная
кислота,

+5  -2

P2O5 + 3H2O →
H6P2O8
2H3PO4 
ортофосфорная
кислота.


 

Классификация
кислот
по
следующим
признакам:

 

По
содержанию
кислорода:

1. Кислородосодержащие                      
2. Бескислородные

HNO3, H2SO4, H3PO4                           
HCl, HJ, H2S

 

По основности

(по числу
атомов
водорода в
молекуле
кислоты)

 

1. Одноосновные 
                                 
2. Многоосновные

HCl, HNO3, HBr    
                               
H2SO4, H3PO4, H2S

 

По силе (по
степени
диссоциации)

 

1. Сильные (α =100 %)   
                       
2. Слабые (α < 100 %)

HCl, HNO3, H2SO4                                  
H2S, HNO2, H2CO3

 

По
растворимости

 

1. Растворимые   
                                  2. Нерастворимые

HCl, HNO3     
                                       H2SiO3, H2MoO4

 

 

Получение

 

1. Бескислородные
кислоты
получают
синтезом из
простых веществ
летучих
соединений с
последующим
растворением
их в воде.
Например:

H2 + Cl2
2HCl хлороводород,
газ.
Растворяем
его в воде,
получаем HCl хлороводородную
кислоту
жидкость.

 

2.
Растворение
соответствующего
оксида в воде:

 

P2O5 + 3H2O →
2H3PO4.

 

3.
Электролиз
растворов
солей:

 

Na2SO4 + 4H2O  H2SO4
+2H2+O2+2NaОН.

 

4.
Взаимодействие
растворимой
соли с сильной
кислотой
(получают
нерастворимые,
легколетучие,
слабые
кислоты):

 

Na2SiO3 + 2HCl → 2NaCI
+ H2SiO3

                                                
            H2O

Na2CO3
+ 2HCl → 2NaCI + H2 CO3

CO2

 

Физические
свойства

 

Большинство
неорганических
кислот жидкости,
смешивающиеся
с водой в
любых соотношениях,
затвердевающие
при низких
температурах;
фосфорная
кислота —
кристаллическое,
похожее на
лед вещество,
хорошо
растворяется
в воде. Кремниевая
кислота
твердое
вещество,
нерастворимое
в воде.
Некоторые
кислоты
существуют
только в
растворе H2Cr2O7, HMnO4.
Их гидратированные
анионы
окрашены в
характерные
цвета: оранжевый,
фиолетовый.
Наконец,
такие
кислоты, как хлороводородная, 
бромоводородная
летучие,
поэтому
обладают
резким
запахом.
Кислоты
имеют кислый
вкус.

 

Химические
свойства

 

1.
Изменение
цвета
индикаторов:

фенолфталеин
бесцветный,

лакмус
розовый,

метилоранж
красный.

2.
Взаимодействие
с
основаниями
с образованием
соли и воды
(реакция
нейтрализации):

 

H2SO4 + 2NaOH →
Na2SO4 + 2H2O.

 

3.
Взаимодействие
с основными
оксидами:

 

H2SO4 + Na2O →
Na2SO4 + 2H2O.

 

4.
Взаимодействие
с солями:

а)
менее
летучие
кислоты
вытесняют
более летучие
из их солей:

H2SO4 + NaCI
→ NaHSO4 + HCI


   конц.                 тверд.

б)
более
сильные
кислоты
вытесняют
менее сильные
из растворов
их солей:

 

3HCI + Na3PO4
3NaCI + H3PO4.

 

         
5.
Взаимодействие
с металлами
различных
кислот
протекает
согласно
положению
металлов в
ряду
напряжений,
который
характеризует
окислительно-восстановительную
способность
электрохимической
системы
металл — ион
металла.

 

Li  К  Ca   Мg  Al   Ti  Cr  Zn  
Fe   Ni   Sn   Pb  H  Cu  Ag  Au 

 

Исходя
из этого, все
металлы
удобно
разделить на
три условные
группы:

 

Активные

Средней

активности

Малоактивные

Li  К  Ca  Мg  Al

Ti Cr  Zn   Fe  
Ni   Sn Pb

Cu   Ag  Au

 

 

 

Взаимодействие
металлов с хлороводородной 
кислотой

.

Активные

Средней
активности

Малоактивные

Реагируют

Реагируют

 (пассивируется
Pb)

Не
реагируют

 

Например:

Zn +2HCI → ZnCI2 + H2

Щелочные
и
щелочноземельные
металлы с растворами
кислот
взаимодействуют
в две стадии:

Na + HCI NaCI + H2

1).2Na + 2H2O →
2NaОН + H2

               
        
2). NaОН
+ HCI→ NaCI + H2O.

 

Малоактивные
металлы,
расположенные
в ряду
напряжений
правее
водорода, из
разбавленного
раствора
кислоты его
не вытесняют:

Cu + HCl

 

Взаимодействие
металлов с
разбавленной
серной
кислотой

 

 

Активные

Средней
актив.

Малоактивные

Реагируют

Реагируют

(пассивируется
Pb)

Не
реагируют

 

 

Например:

Zn + H2SO4 → ZnSO4
+ H2

Малоактивные
металлы,
расположенные
в ряду
напряжений
правее
водорода, из
разбавленного
раствора
кислоты его
не вытесняют:

Cu + H2SO4

 

Взаимодействие
металлов с
концентрированной
серной 
кислотой

 

Активные

Средней
активности

Малоактивные

Реагируют

Реагируют

(пассивируются
Al, Fe)

Реагируют
Сu, Hg

не
реагируют Ag,

Au, Pt

 

В
результате
взаимодействия
образуются сульфат
металла,     
вода и один
из продуктов
окисления
серной
кислоты:

 

S ,   H2S,  SO2

S ,   H2S,  SO2

           
SO2

 

Например:      Zn +
2H2SO4(К) → ZnSO4 + SO2 + 2H2O,

4Zn + 5H2SO4(К) → 4ZnSO4 + H2S +
4H2O,

  
                  3Zn
+ 4H2SO4(К) → 3ZnSO4 + S + 4H2O,

            2H2SO4(к) + Сu → Сu SO4 + SO2 + 2H2O.

 

Холодная
концентрированная
серная кислота
пассивирует
Al, Fe; при
нагревании пассивирующие
пленки
растворяются,
и
взаимодействие
с кислотой
протекает
интенсивно.

 

Взаимодействие
металлов с
разбавленной
азотной
кислотой

 

 

Активные

Средней
активности

Малоактивные

Реагируют

Реагируют

(пассивируется
Ti)

Реагируют
Сu, Hg Ag,

не
реагируют

Au, Pt

 

В
результате
взаимодействия
образуются нитрат
металла,
        вода
и один из
продуктов
окисления
азотной кислоты:

 

  NH3,    
NH4NO3

      
N2
или N2O

           
NO

 

Например:

 

10HNO3 + 4Mg → 4Mg(NO3)2
+ NH4NO3 + 3H2O.

             

 

Взаимодействие
металлов с
концентрированной
азотной
кислотой

 

 

Активные

Средней
активности

Малоактивные

Реагируют

Реагируют

 (пассивируются
Ti, Al, Cr, Fe)

Реагируют
Сu, Hg Ag,

не
реагируют

Au, Pt

 

В
результате
взаимодействия
образуются нитрат
металла, вода
и NO2
(газ бурого
цвета). При
взаимодействии
с кислотой
активных
металлов
возможно
выделение  N2O.

 

Например:

 

4HNO3 + Сu → Сu (NO3)2 + 2NO2 +
2H2O.

10HNO3 + 4Са → 4Са (NO3)2 + N2O + 5H2O.

 

При
взаимодействии
азотной
кислоты любой
концентрации
и
концентрированной
серной с
металлами водород
никогда не
выделяется
.

Холодная
концентрированная
азотная
кислота пассивирует
следующие
металлы Fe, Cr, Al, Ti, но при
нагревании
взаимодействие
этих металлов
с кислотой
протекает
энергично.

6.
Взаимодействие
с
неметаллами
азотной и концентрированной
серной
кислот:

 

3P + 5HNO3 + 2H2O → 3H3PO4
+ 5NO,

C + 2H2SO4(к)
→ CO2 + 2SO2 + 2H2O.

 

 

7.
При
нагревании
некоторые
кислоты
разлагаются:

H2SiO3 H2O + SiO2.

 

 

 

 

 

 

    
Ход работы:

 

Опыт
№ 1.
Взаимодействие
кислотного
оксида

с
водой

 

Стеклянной
палочкой
взять
кусочек
оксида
фосфора (V) и
смешать его с
небольшим
количеством
воды. В
полученный
раствор
добавить 23
капли лакмуса.
Написать
уравнение
реакции.

 

 

Опыт
№ 2.
Взаимодействие
соли с
кислотой

 

Положить
в пробирку
немного
сухого ацетата
натрия

CH3COONa
или другой
соли
уксусной
кислоты и
добавить
небольшое
количество
разбавленной
серной
кислоты.
Образовавшаяся
кислота
может быть
обнаружена
по запаху. При
выяснении
запаха нельзя
нюхать
вещество
прямо из
пробирки, так
как вдыхание
газов и паров
может вызвать
сильное
раздражение
дыхательных
путей. Необходимо
ладонью руки
сделать
легкие движения
от пробирки к
носу.
В этом
случае в нос
будет
попадать
смесь газа с
воздухом и
сильного
раздражения
не
произойдет.

В
пробирку
внести 56
капель
концентрированного
раствора
силиката
натрия Na2SiO3
и постепенно
добавить 56
капель
соляной кислоты,
непрерывно
встряхивая
пробирку. Образуется
студнеобразный
осадок.
Подействовать
на
полученный
осадок
раствором
щелочи и
серной
кислоты. Отметить
изменения в
пробирках и
написать
уравнения
реакций.

 

 

Опыт
№ 3.
Взаимодействие
основного
оксида с
кислотой

 

К
небольшому
количеству
оксида
магния в пробирке
прилить
небольшое
количество
соляной
кислоты.
Отметить
растворение
осадка.
Написать уравнения
реакции.
Какая соль
образовалась?

 

Опыт
№ 4.
Взаимодействие
основания с
кислотой

(реакция
нейтрализации)

 

Налить в
фарфоровую
чашку 10 см3 2
моль/дм3
раствора гидроксида
натрия.
Добавить 12
капли
фенолфталеина,
чтобы
окраска
стала
малиновой,
что подтверждает
наличие
щелочи в
растворе. Затем
в пробирку по
каплям
добавлять
раствор
соляной
кислоты, до
обесцвечивания
раствора.
Выпарить
содержимое
чашки и
убедиться,
что
образовалась
соль.
Написать
уравнение реакции.

Страница не найдена — Портал Продуктов Группы РСС

Сообщите нам свой адрес электронной почты, чтобы подписаться на рассылку новостного бюллетеня. Предоставление адреса электронной почты является добровольным, но, если Вы этого не сделаете, мы не сможем отправить Вам информационный бюллетень. Администратором Ваших персональных данных является Акционерное Общество PCC Rokita, находящееся в Бжег-Дольном (ул. Сенкевича 4, 56-120 Бжег-Дольный, Польша ). Вы можете связаться с нашим инспектором по защите личных данных по электронной почте: .

Мы обрабатываем Ваши данные для того, чтобы отправить Вам информационный бюллетень — основанием для обработки является реализация нашей законодательно обоснованной заинтересованности или законодательно обоснованная заинтересованность третьей стороны – непосредственный маркетинг наших продуктов / продуктов группы PCC .

Как правило, Ваши данные мы будем обрабатывать до окончания нашего с Вами общения или же до момента, пока Вы не выразите свои возражения, либо если правовые нормы будут обязывать нас продолжать обработку этих данных, либо мы будем сохранять их дольше в случае потенциальных претензий, до истечения срока их хранения, регулируемого законом, в частности Гражданским кодексом.

В любое время Вы имеете право:

  • выразить возражение против обработки Ваших данных;
  • иметь доступ к Вашим данным и востребовать их копии;
  • запросить исправление, ограничение обработки или удаление Ваших данных;
  • передать Ваши персональные данные, например другому администратору, за исключением тех случаев, если их обработка регулируется законом и находится в интересах администратора;
  • подать жалобу Президенту Управления по защите личных данных.

Получателями Ваших данных могут быть компании, которые поддерживают нас в общении с Вами и помогают нам в ведении веб-сайта, внешние консалтинговые компании (такие как юридические, маркетинговые и бухгалтерские) или внешние специалисты в области IT, включая компанию Группы PCC .

Больше о том, как мы обрабатываем Ваши данные Вы можете узнать из нашего Полиса конфиденциальности.

Аккумуляторы VARTA® — наши советы по безопасной установке и обращению

← Основная информация об аккумуляторах

Устанавливая батарею или работая с ней, соблюдайте свою безопасность и безопасность Ваших коллег, выполняя все инструкции безопасности и советы, указанные внизу.

ОПАСНОСТЬ ВЗРЫВА БАТАРЕЙ

Батареи содержат серную кислоту, в них образуются взрывоопасные смеси водорода и кислорода.

При работе с батареей или поблизости с ней всегда надевайте защитные очки, защитную маску или брызгозащитные очки по стандарту ANSI Z87.1 (U.S.) или CE EN166 (Европа). Всегда надевайте соответствующую защиту глаз, лица и рук. Не допускайте попадания искр, пламени или сигарет на батарею. Никогда не пытайтесь открыть батарею с несъемными пробками. (На рис. 1 указаны обозначения и символы, используемые на колпачках вентиляционных отверстий).

Съемные пробки должны быть прочно и ровно посажены при обслуживании электролита.

Помещение, в котором проводятся работы, должно хорошо проветриваться.

Никогда не наклоняйтесь над батареей во время повышения напряжения, проверки или зарядки.

Проявляйте большую осторожность во время работы с металлическими инструментами или проводниками во избежание короткого замыкания и искрообразования

БЕЗОПАСНАЯ ЗАРЯДКА

НИКОГДА НЕ ПЫТАЙТЕСЬ ЗАРЯЖАТЬ БАТАРЕЮ, НЕ ПРОЧИТАВ ПЕРЕД ЭТИМ ИНСТРУКЦИИ ПО ЭКСПЛУАТАЦИИ ИСПОЛЬЗУЕМОГО ЗАРЯДНОГО УСТРОЙСТВА. Кроме инструкций изготовителя зарядного устройства также должны выполняться следующие общие меры предосторожности:

Всегда надевайте соответствующую защиту глаз, лица и рук.

Всегда заряжайте батареи в хорошо проветриваемом помещении.

Всегда заряжайте батарею в хорошо проветриваемом помещении.

Пробки должны быть посажены прочно и ровно.

Выключите зарядное устройство и таймер перед соединением проводов к батарее во избежание опасных искр.

Никогда не пытайтесь заряжать батарею, которая очевидно повреждена или замерзла.

Соедините провода зарядного устройства с батареей; красный положительный (+) провод с положительным (+) выводом и черный отрицательный (-) провод с отрицательным (-) выводом. Если батарея все еще находится в транспортном средстве, соедините отрицательный провод с корпусом двигателя, служащим заземлением.

Убедитесь в том, что зажигание и все электрические аксессуары выключены. (Если двигатель имеет положительное заземление, соедините положительный провод с корпусом двигателя).

Убедитесь в том, что провода зарядного устройства, подводящиеся к батарее, не повреждены или не изношены. Установите таймер, включите зарядное устройство и медленно повышайте величину тока заряда до достижения желаемого показателя в амперах.

Если батарея становится горячей или выделяется электролит в виде газа или брызг, уменьшите величину тока заряда или временно выключите зарядное устройство. Всегда выключайте зарядное устройство перед отсоединением его проводов от батареи во избежание опасных искр.

РАБОТА С ЭЛЕКТРОЛИТОМ БАТАРЕИ

Электролит представляет собой раствор серной кислоты и воды, который может повредить одежду и вызвать ожог кожи. БУДЬТЕ ЧРЕЗВЫЧАЙНО ОСТОРОЖНЫ ПРИ РАБОТЕ С ЭЛЕКТРОЛИТОМ и держите наготове раствор для нейтрализации кислоты — например, водный раствор пищевой соды или нашатырного спирта. При работе с батареей:

Всегда надевайте соответствующую защиту глаз, лица и рук.

Если электролит попал в глаза, сразу откройте глаза и промойте их чистой, прохладной водой не менее 15 минут. Сразу обратитесь за медицинской помощью. Если электролит попал внутрь организма, выпейте воды или молока в больших количествах. НЕ пытайтесь вызвать рвоту. Сразу обратитесь за медицинской помощью.

Нейтрализуйте все брызги, попавшие на транспортное средство или рабочую площадь, пищевой содой. После нейтрализации промойте дочиста загрязненную область водой. Чтобы подготовить электролит с конкретной плотностью, всегда медленно лейте концентрированную кислоту в воду. НЕ лейте воду в кислоту. Всегда размешивайте воду, добавляя небольшие порции кислоты. Если она становится горячей, дайте раствору остыть перед добавлением следующей порции кислоты.

Дополнительные ресурсы и загрузки:

Необходимый уход и обслуживание (PDF)

Руководство по тестированию аккумулятора (PDF)

Почему в жаркий летний день нельзя утолять жажду газированной водой?

Др. Кати Нурк, стоматолог, Объединенные клиники Unimed

В жаркие солнечные дни мы зачастую хватаем бутылки со вкусной и газированной водой. Потреблять больше воды в жаркую погоду необходимо, потому что обезвоживание, которое может наступить из-за жары и физической активности, имеет множество неприятных последствий: головная боль, усталость, сонливость, а также опасность потерять создание. Выбрать вместо лимонада воду – это хорошее начинание, но находящиеся в некоторых бутылках с водой добавленный сахар, углекислый газ и вкусовые добавки вредны для наших зубов.

Что на самом деле содержится в газированной воде и лимонаде?

В состав газированной и подслащенной воды входят, как правило, углекислый газ, сахар, различные вкусовые добавки и ароматизаторы. Например, одна бутылка воды со вкусом клубники содержит помимо воды также сироп глюкозы и фруктозы, лимонную кислоту, натуральные вкусовые добавки, а также ароматизаторы и консерванты (E202 и E211). На данном примере видно, что в 1,5 литровой бутылке с воды хитро спрятано 80 г или 20 чайных ложек сахара!

Как добавленные сахара влияют на наши зубы?

Сахар, который добавляют в воду для получения приятного и сладкого вкуса, дает необходимую пищу для бактерий, живущих на зубах. Из-за этого наши зубы становятся уязвимыми. После каждого приема пищи зубы подвергаются кислотной атаке, которую наш организм в состоянии нейтрализовать, но если количество приемов пищи и потребление сладкой воды в течение дня слишком большое, то процессы, которые разрушают зубы, не прекращаются, и так в зубах образуются дырки.

Как углекислый газ и кислотные добавки влияют на наши зубы?

Добавляемый в напитки углекислый газ повышает уровень кислотности в воде, что означает, что теоретически газированная вода при длительном воздействии может растворить зубы, поскольку эмаль — это минеральное образование. Но исследования доказали, что употребление газированной (минеральной) воды в умеренных количествах не опасно для зубов, потому что слюна может нейтрализовать кислоту.

Но если в газированную воду добавить кислотные вкусовые добавки, например, лимонную кислоту или другие фруктовые добавки, то уровень pH станет еще ниже. Это увеличивает эрозионное воздействие на зубы. Другими словами, зубы будут быстрее изнашиваться и могут стать чувствительными. Лимонная кислота — одна из самых вредных для зубов кислот, но и одна из самых распространенных вкусовых добавок в продаваемых напитках.

Какую воду выбрать?

Для наших зубов самое полезное пить воду из-под крана или негазированную минеральную воду. Последняя особенно нужна после тренировки или во время жаркой погоды. С потом тело теряет много нужных солей и, поскольку вода без газов, эрозия зубам не грозит. Некоторые виды минеральной воды почти с базовым pH.

При умеренном употреблении годится и газированная минеральная вода, поскольку, как ранее упоминалось, слюна способна нейтрализовать снижение уровня pH и эрозийное воздействие углекислого газа низкое.

Как можно реже рекомендуется употреблять воду с добавленным сахаром, это может способствовать возникновению дырок в зубах, а также воду с кислотными добавками (например, с лимоном или лаймом).

Как уменьшить негативные последствия воздействия подслащенной воды или воды с кислотными добавками?

Умеренное употребление газированной и подслащенной воды не вызовет сразу же образование дырок в зубах. Важно не количество воды, а как часто вы ее пьете. Чем реже, тем лучше. Особенно внимательными должны быть люди, у которых уже есть эрозийные повреждения зубов или у кого есть предрасположенность к образованию зубных дырок.

Для уменьшения вредного воздействия подслащенной воды или воды с кислотными добавками рекомендуется:

➡️ употреблять напитки только во время приема пищи — для утоления жажды пейте чистую воду!

➡️  после употребления воды с добавками прополоскайте рот чистой водой.

➡️ употребляйте газированную воду теплой — так снижается способность углекислого газа растворяться в воде, и уровень pH воды будет выше.

➡️ после употребления подслащенной воды жуйте жвачку без сахара, она стимулирует выделение слюны. Также подойдет, например, сыр, который помогает нейтрализовать кислоту.

➡️ пейте через трубочку — так контакт напитка с зубами будет меньше.

Рекомендации врачей — Инфекционная больница Павловского


Пятница, 

Сентябрь 
2016

ПИВНОЙ АЛКОГОЛИЗМ

Феномен формирования пивной алкогольной зависимости в общей популяции населения и, особенно, в молодёжной среде, привлёк внимание широких слоёв общественности в последние годы. В некоторых публикациях специалистов в средствах массовой информации даётся крайне пессимистическая оценка возросшего уровня потребления пива среди молодого поколения: предлагаются ограничительные санкции на рекламу пива, введение возрастного ценза на продажу пива, запрет на производство и продажу пива с повышенным содержанием алкоголя.

Итак, что же такое – пиво? Классическое определение понятия пива звучит следующим образом: Пиво – это слабоалкогольный ароматный пенистый напиток с хмелевой горечью, изготовленный путём естественного брожения из проросшего ячменя (солода), хмеля, дрожжей и воды. Современное пиво представляет собой более сложный продукт, где все вышеупомянутые признаки являются относительными. В обычном пивном производстве при повышении концентрации алкоголя свыше 6-7% (об.) трудно обеспечить хорошие вкусовые качества пива. Относительным является и понятие естественного брожения в современном пивоваренном процессе. Можно считать весьма условным использования только 4-х компонентов при производстве пива (ячменя (солода), хмеля, дрожжей и воды). В настоящее время при производстве пива разрешено использовать целый ряд добавок: не преобразованный в солод ячмень, кукурузу, рис, пшеницу и просо, которые могут составлять до 40% применяемых зерновых; уксусная, молочная, фосфорная и серная кислота могут добавляться в воду для варки; хлорид цинка, железа и аммония могут использоваться как добавки в сусло; сахарозу, глюкозу или изменённый сахар можно добавлять в сусло и они могут составлять до 20%; танин, агар – агар, кислоты и красители разрешается добавлять для стабилизации сусла; вещества против пенообразования, улучшающие качество пены, антиокислители и ферменты  могут быть добавлены в бродильные сосуды; стабилизаторы, консерванты можно применять при подготовке продукта к хранению. Сахарин и солодовый сахар добавляют для придания сладковатого привкуса. Чем опасны данные химические компоненты? Нам кажется, что не имеет смысла объяснять действие серной кислоты на организм человека. Диоксид серы и серная кислота раздражающе действует на слизистую желудка. Использование ферментов приводит к резкому возрастанию риска возникновения аллергических реакций. Стабилизаторы мутности могут привести к возникновению гастритов и энтеритов, краситель татразин – аллергической реакции. Используемые сивушные масла вызывают тяжёлую сомато-вегетативную реакцию с головной болью, слабостью, гипергидрозом, сердцебиением. Используемые для дезинфекции тары формальдегид способен вызывать раковые заболевания. Таким образом, некоторые сорта консервированного пива представляют собой своеобразный «химический коктейль» несущий в большей степени угрозу здоровью, нежели пользу от его применения.

ОБЩЕЕ ВЛИЯНИЕ ПИВА НА ОРГАНИЗМ ЧЕЛОВЕКА

Пивной алкоголизм формируется медленнее водочного, более незаметно и обманчиво. Для него очень характерно наличие у человека отрицания заболевания и отсутствие критики к своему состоянию. Еще одна особенность — быстрое развитие выраженной психической зависимости. Если человек выпивает кружку пива в день, если он чувствует психологический дискомфорт оттого, что не выпил — это уже пивная зависимость. Из-за вкуса, наличия седативных компонентов и небольших градусов бороться с влечением к пиву сложнее, чем с влечением к водке. Постоянное потребление пива влияет на интеллект человека, снижает его способность к обучению, приводит к безжалостному вымыванию из организма белков, микроэлементов, витаминов. В результате, дефицит магния приводит к изменению фона настроения, человек плохо спит, становится раздражительным. От вымывания витамина С страдает интеллект, снижается иммунитет, чаще возникают простудные заболевания. Еще в XVI веке немецкие врачи к группе специфических «пивных» болезней, таких как «пивное сердце» (стойкое расширение полостей сердца, утолщение его стенки, при этом учащается частота сердечных сокращений, возникают аритмии, повышается давление) и «пивной живот» отнесли так же и «пивную травму» (разрыв мочевого пузыря при избыточном пивопотреблении). Физиологически большой пивной живот – это слабость мышц брюшного пресса, их пониженный тонус, приводящий в последствии к ухудшению перистальтики кишечника, запорам, образованию жировых отложений в области талии, патологическому смещению органов брюшной полости: печени, почек, желудка, кишечника и грудной клетки: сердца и диафрагмы. Вследствие чего возрастает риск нарушения нормального кровоснабжения вплоть до возникновения участков ишемии и инфарктов, варикозному расширению вен.

Профилактика алкоголизма

Пивной алкоголизм, как и многие другие болезни, можно предупредить. На это нацелена профилактика алкоголизма в России, в каждой отдельной семье, в каждом учебном учреждении, на каждом предприятии. Профилактика выражается прежде всего в формировании у населения здорового образа жизни, при котором употребление алкоголя становится неприемлемым.

Профилактика алкоголизма в России на государственном уровне заключается в следующем:

·         запрет на продажу алкогольных напитков лицам, не достигшим совершеннолетия;

·         уменьшение количества торговых точек продажи алкоголя и мест для его употребления;

·         запрет на рекламу алкоголя;

·         формирование негативного отношения к алкоголю, отсутствие поощрения посиделок после работы;

·         контроль за качеством продаваемой алкогольной продукции;

·         уголовная и административная ответственность за появление граждан в состоянии алкогольного опьянения в общественных местах;

·         применение индивидуальных профилактических мер на производстве.

Кроме того, для приобщения людей к здоровому образу жизни и занятиям спорта строятся спортивные объекты, открываются различные спортивные секции и залы.

 

 

Какую кислоту заливают в аккумулятор

Большинство водителей никогда в жизни не задумывались, какую кислоту добавляют в аккумулятор авто и зачастую высказывают неправильные догадки. Одни скажут, что там вода, другие назовут соляную кислоту, поэтому мы расставим все точки над «и». В аккумуляторах применяется смесь серной кислоты и дист. воды. Знакомьтесь, это электролит.

На практике встречаются устройства, где в роли электролита выступает щелочь (никель-кадмиевые аккумуляторы). Также есть несколько гелевых устройств, но по факту гель – это та же самая кислота, находящаяся в другом состоянии или пропитывающий стекловолоконные частицы.

Она применяется в промышленном изготовлении АКБ для авто. Обычно, электролит — это 35% серной кислоты и 65% дист. воды. Заливать любую другую воду категорически запрещается, поскольку она содержит соли, которые существенно сократят срок работы АКБ.

Концентрированная серная кислота производится в 2 стадии. В первой её концентрацию доводят до 65-70%, а во второй – до 98%. Это необходимо для того, чтобы увеличить срок хранения кислоты. Вероятно даже повышение концентрации до 99%, но из-за потерь SO3 значение все равно будет снижено до 98 процентов.

Как сделать электролит в домашних условиях?

Рекомендуем купить его в магазине, но его можно сделать и своими руками.

Нам потребуются:                                                      

  • Дистиллированная вода;
  • Серная кислота;
  • Емкость из стекла, керамики или свинца, устойчивая к кислотам;
  • Эбонитовая палочка.

Чтобы сделать электролит надо залить воду в специальную емкость и добавить туда серную кислоту в пропорции 3:1. Тщательно перемешать смесь эбонитовой палочкой. Вещество плотно накрыть и оставить на 1 день до выпадения осадка.

Таблица  для расчета приготовляемого электролита

Необходимо сказать, что превышения в допустимой концентрации электролита может привести к самым неприятным последствиям. От чрезмерно концентрированного вещества пластины аккумулятора очень быстро «разъест», что в кратчайшие сроки вызовет выход аккумулятора из рабочего состояния.

В дист вода добавляем кислоту, а не наоборот

А вот слишком низкий уровень плотности вещества может очень быстро вызывать сульфатацию. Наверняка вы знаете, что это такое. Нет? Это процесс оседания кристаллических осадков на пластинах, в результате чего аккумулятор теряет свои свойства и просто оказывается неспособным держать заряд долгое время.

Зачем нужен электролит?

Суть работы АКБ базируется на реакциях, проходящих только при помощи этого вещества. Батарея состоит из плюсовых и минусовых пластинок, погружаемых в кислотный раствор. Эти пластины оснащены токоотводными решетками, сделанными из свинца.

Как проконтролировать состояние электролита?

Держать под контролем состояние электролита нужно. Для контроля сгодится пластмассовый корпус от обычной пластиковой ручки. Открываем пробки и погружаем его к пластинам, после чего зажимаем сверху и вынуть. Идеальный уровень – 10-12 мм. Если надо, дольем электролит.

Измерение уровня электролита в АКБ

Очень важно следить за временем года и параметрами раствора в соответствии с ним. Можно существенно продлить жизнь батареи, если использовать растворы с разными параметрами в разное время. К примеру, слишком концентрированное вещество в летнее время деструктивно воздействует на АКБ. Высокая плотность увеличивает энергопотери, а серный ангидрид может полностью разрушить электроды устройства.

Срок эксплуатации электролита

У такого электролита нет срока эксплуатации. Данный критерий может быть определен только по способности выполнения своих функций на 100%. Среди факторов, которые могут повлиять на жизненный цикл электролита, важнейшими являются:

  • Плотность;
  • Температуру работы аккумулятора;
  • Заряд.

Если все показатели в норме, то и электролит не будет тревожить Вас годами.

Как увеличить плотность вещества?

Увеличить плотность можно при повышении температуры и гидролиза. Также важно постоянно добавлять дистиллят для поддержания показателей на нормальном уровне. Если же концентрация кислоты ниже 1,275 грамм на сантиметр кубический, её нужно поднять.

Корректировка плотности электролита в АКБ

Повысить такой уровень можно 2-мя способами: сменой содержимого или доливом концентрированной кислоты.

Если разбавлять жидкость, то необходимо совершить определенные действия для всех банок:

  • Откачать максимум электролита при помощи шприца;
  • Внести в банку половину от его объема;
  • Подать нагрузку с низкой мощностью для смешивания жидкости.
  • Если изменений в уровне нет, добавляем во вторую половину электролит и получаем идеальную плотность.

Кислота – вредная субстанция для кожи. Использование резиновых перчаток, фартука и защитных очков при работе с этим веществом категорически обязательно!

Как заливать и доливать кислоту?

Электролит заливается при помощи трубки до 15 мм над пластинками батареи. Затем оставляем аккумулятор на 2 часа для остывания и заряжаем током в 10 раз меньшим от объема АКБ.

Ключевые характеристики надо проверять пару раз в году с помощью особого прибора и не пропускать проверки.

При смешивании дистводы с кислотой, важно добавлять именно кислоту в воду, так как в обратном случае есть вероятность активации бурной реакции вещества с тепловыделением, доведением её до состояния кипения.

Внимательно соблюдайте все вышеперечисленные рекомендации и АКБ будет служить очень долгое время!

Сдать старый АКБ за деньги вы сможете в нашей фирме, тарифы на приём акумуляторов на этой странице.

02.0 Кислотно-щелочная нейтрализация

2.1 Кислоты

Если вы хотите разбавить кислоту водой перед нейтрализацией ее основанием (например, гидроксидом натрия, гидроксидом калия или бикарбонатом натрия), всегда добавляйте кислоту в воду; никогда не добавляйте воду в кислоту .

Выполняйте все нейтрализации в вытяжном шкафу, надев перчатки из нитриловой резины, лабораторный халат и защитные очки.

2.1.1 Соляная кислота

  1. Медленно добавьте соляную кислоту в емкость с холодной водой, чтобы получить раствор кислоты в воде 1:10.
  2. Медленно добавляйте 1M раствор гидроксида калия, гидроксида натрия или карбоната натрия до тех пор, пока pH не станет в диапазоне от 6,0 до 8,0.
  3. Смыть в канализацию избытком холодной воды.

2.1.2 Серная кислота

  1. Медленно добавьте серную кислоту в емкость с ледяной водой, чтобы получить раствор кислоты в воде 1:10.
  2. Медленно добавляйте карбонат натрия, пока pH не станет в диапазоне от 6,0 до 8,0.
  3. Смыть в канализацию избытком холодной воды.

2.1.3 Уксусная кислота

  1. Медленно добавьте уксусную кислоту в емкость с холодной водой, чтобы получить раствор кислоты в воде 1:10.
  2. Медленно добавляйте 1М раствор гидроксида натрия или карбоната натрия, пока pH не станет в диапазоне от 6,0 до 8,0.
  3. Смыть в канализацию избытком холодной воды.

2.1.4 Фосфорная кислота

  1. Медленно добавьте фосфорную кислоту в емкость с холодной водой, чтобы получить раствор кислоты в воде 1:10.
  2. При перемешивании медленно добавляйте карбонат натрия, пока pH не станет в диапазоне от 6,0 до 8,0.
  3. Смыть в канализацию избытком холодной воды.

2.2 Базы

2.2.1 Гидроксид калия

  1. При перемешивании медленно добавьте гидроксид калия в емкость с ледяной водой, чтобы получить раствор основания к воде 1:10.
  2. Медленно добавляйте 1 М соляную кислоту примерно по 1 мл за раз, пока pH не станет между 6,0 и 8,0.
  3. Смыть в канализацию избытком холодной воды.

2.2.2 Гидроксид натрия

  1. При перемешивании медленно добавьте гидроксид натрия в емкость с ледяной водой, чтобы получить раствор основания к воде 1:10.
  2. Медленно добавляйте 1 М соляную кислоту примерно по 1 мл за раз, пока pH не станет между 6,0 и 8,0.
  3. Смыть в канализацию избытком холодной воды.

2.2.3 Гидроксид кальция

  1. При перемешивании медленно добавьте гидроксид кальция в емкость с ледяной водой, чтобы получить раствор основания к воде 1:10.
  2. Медленно добавляйте 1 М соляную кислоту примерно по 1 мл за раз, пока pH не станет между 6,0 и 8,0.
  3. Смыть в канализацию избытком холодной воды.

Урок по кислотам, щелочам и водным наукам

При растворении в воде кислоты отдают ионов водорода (H +) . Ионы водорода — это атомы водорода, которые потеряли электрон и теперь имеют только протон, что дает им положительный электрический заряд. Основания, с другой стороны, смешанные с водой дают гидроксид-ионов (ОН-) .Если раствор имеет высокую концентрацию ионов H +, то он кислый. Если раствор имеет высокую концентрацию ионов ОН-, то он является основным.

Во многих кислотно-щелочных реакциях образуются вода и соль. Если объединить соляную кислоту (HCL) и основной гидроксид натрия (NaOH), то получится H 2 O (вода) и NaCl (хлорид натрия, поваренная соль). Ионы H + в кислоте присоединяются и нейтрализуются ионами OH- основания с образованием H 2 O.

Одним из самых простых способов показать, как кислоты и основания взаимодействуют друг с другом (и продемонстрировать их различные свойства), является создание «вулкана» пищевой соды и уксуса . Чтобы вызвать сильную сыпь, используйте небольшую пластиковую бутылку (подойдут безалкогольные напитки объемом 20 унций). Наполните бутылку наполовину (от 1 до 1,5 стакана) уксусом. Чтобы вызвать сыпь, бросьте в бутылку «бомбу» из пищевой соды — оберните одну столовую ложку пищевой соды в небольшой кусочек папиросной бумаги, завязав концы ниткой.Вы должны увидеть мгновенное извержение! Пищевая сода нейтрализует кислоту в уксусе. При этом выделяется углекислый газ, который вызывает шипение в вашем вулкане. (Кислый раствор нейтрализуется при добавлении к нему основания, а щелочной раствор нейтрализуется добавлением кислоты.)

Для другого эксперимента с реакцией поместите таблетку Alka-Seltzer на дно канистры из прозрачной пластиковой пленки (такой, где крышка помещается внутри, а не закрывается снаружи).Наполните канистру теплой водой, а затем быстро закройте крышку и наблюдайте за кислотно-щелочной реакцией!

Шкала pH используется для измерения количества ионов H + в растворе. Кислоты имеют pH ниже 7; основания имеют pH выше . Сильные кислоты имеют самый низкий уровень pH (0-4), а сильные основания — самые высокие уровни pH (10-14). Нейтральные растворы имеют pH 7, и они не являются ни кислыми, ни основными. Дистиллированная вода нейтральна, потому что ионы H + и OH- сбалансированы.

Litmus — это природный кислотно-щелочной индикатор, получаемый из одного вида лишайников.Если у вас есть красная и синяя лакмусовая бумага, вы можете протестировать разные растворы на предмет того, являются ли они кислотами или щелочами. Синяя лакмусовая бумажка становится красной , когда раствор является кислым; красная лакмусовая бумажка превращается в синюю в в базовых решениях. Попробуйте протестировать средство для мытья окон, средство для чистки унитаза, апельсиновый и яблочный сок — налейте немного каждого в отдельные пробирки, небольшие стаканы или баночки. Используйте лакмусовую бумажку, чтобы определить, какие кислоты, а какие — основания. Вот уровни pH некоторых других веществ, которые вы можете проверить: лимонного сока (2), уксуса (3), молока (6), яичных белков (8), пищевой соды (9) и аммиака (10).Кровь человека имеет идеальный pH 7,4; даже небольшие колебания могут серьезно повлиять на наш организм.

Вы также можете сделать свой собственный индикатор pH. — с помощью блендера смешайте одну часть измельченной красной капусты с двумя частями кипящей воды и используйте сок для тестирования различных растворов. Кислоты превращают пигменты в индикаторе в красноватый цвет; основы сделают пигменты голубоватыми или желто-зелеными.

Урок естествознания: хорошая вода

Вода иногда может казаться простой и скучной, но у нее есть некоторые интересные физические свойства! Вода в точке замерзания весит около 62 фунтов на кубический фут; замерзает при 0 ° по Цельсию и закипает при 100 °; и составляет около 60% нашего тела и 83% нашей крови! Что касается того, на что она способна, то воду называют «универсальным растворителем », потому что она может растворять больше веществ, чем любая другая жидкость, даже самая сильная кислота.

Химический состав воды: H 2 O — два атома водорода и один кислород. То, как эти атомы соединяются вместе, образуя молекулу воды, придает воде особые свойства. Два атома H образуют слабые водородные связи с кислородом; они прикрепляются к вершине молекулы, как уши Микки Мауса.

Эта молекулярная структура придает молекуле воды полярность или однобокий электрический заряд, который притягивает другие атомы.Конец молекулы с двумя атомами H заряжен положительно. Другой конец, с кислородом, заряжен отрицательно. Так же, как в магните, где северные полюса притягиваются к южным полюсам (противоположности притягиваются), положительный конец молекулы воды будет соединяться с отрицательным концом других молекул.

Вода — единственное природное вещество, которое может существовать во всех трех состояниях материи — твердом, жидком и газообразном — при температурах, обычно встречающихся на Земле. Многие другие вещества необходимо перегреть или охладить, чтобы изменить состояние.Твердое состояние воды (то есть лед) менее плотное, чем жидкое состояние, поэтому лед плавает.

Вещества Polar легко растворяются в воде, «универсальном растворителе». Это потому, что один конец полярной молекулы заряжен более положительно, а один конец — более отрицательно. Положительно заряженный конец полярной молекулы будет притягиваться к отрицательно заряженному концу молекулы воды, и наоборот. Точно так же положительные ионы притягиваются к отрицательному концу молекулы воды и наоборот.Это молекулярное притяжение позволяет такому количеству веществ легко растворяться в воде — молекулы окружены молекулами воды. Масло и другие неполярные растворы не имеют положительно или отрицательно заряженных концов, поэтому молекулы воды будут притягиваться друг к другу, а не к маслу. Вот почему масло не смешивается с водой.

Выдающийся ученый: сэр Хэмфри Дэви (1778-1829)

Дэви родился в графстве Корнуолл, Англия, 17 декабря 1778 года.Он получил там образование и стал учеником хирурга-аптекаря, когда умер его отец. Через несколько лет он переехал в Бристоль и стал суперинтендантом медицинского учреждения. В возрасте 22 лет он написал Researches, Chemical and Philosophical о своей работе по воздействию закиси азота (позже известной как веселящий газ). Он экспериментировал с газом, даже сам вдыхал его — одна из предполагаемых причин болезни, от которой он страдал к концу своей жизни.

Следующая его работа была в области электрохимии.Его использование электролиза (пропускание электрического тока через вещество, чтобы разбить его на части) привело его к правильному предположению о том, что элементы в соединении удерживаются вместе электрическими силами.

Дэви выделил элементы натрий, магний, калий, кальций, бор (вместе с Гей-Люссаком) и барий. Он также определил, что хлор является элементом, а не кислородным соединением, и дал ему свое название (от греческого слова «желто-зеленый»). Он экспериментировал с йодом, доказал, что алмазы образованы углеродом, и справедливо предположил, что кислоты содержат водород.В целом Дэви выделил больше элементов, чем любой другой химик до 90–169-х годов -го века.

Хотя он был известен своими работами в области химии, Дэви был более известен своим изобретением безопасной шахтерской лампы. Газообразный метан в угольных шахтах взорвался бы при контакте с пламенем свечей, которые шахтеры использовали для освещения, что привело бы к гибели людей и ущербу. Дэви попросили найти решение. Он придумал дизайн лампы, которая окружала пламя тонкой проволочной сеткой. Некоторое количество света все еще могло светить, но вызванное пламенем только содержало взрывы в камере, где газ попал в лампу; марля не позволяла теплу пламени распространиться на внешние газы.Метод Дэви был использован в более поздних усовершенствованиях освещения шахт.

За свою жизнь Дэви был удостоен множества наград. Он был избран членом Королевского общества в 1803 году; два года спустя он получил престижную медаль Копли; а в 1820 году он был избран ее президентом. Кроме того, он получил награду от Наполеона Бонапарта, которого впечатлила его работа в области электрохимии. −_ {(aq)}} \ label {Eq1} \]

В других случаях молекула воды может отдавать протон и, таким образом, действовать как кислота Бренстеда-Лоури .−_ {(водн.)} + NH_ {3 (водн.)} \ Label {Eq2} \]

В данном случае NH 2 представляет собой основание Бренстеда-Лоури (акцептор протона).

Итак, в зависимости от обстоятельств, H 2 O может действовать как кислота Бренстеда-Лоури или как основание Бренстеда-Лоури. Вода — не единственное вещество, которое может реагировать как кислота в одних случаях или как основание в других, но это, безусловно, самый распространенный пример — и самый важный. Вещество, которое может отдавать или принимать протон, в зависимости от обстоятельств, называется амфипротическим соединением .−_ {(aq)} \ label {Eq3} \]

Этот процесс называется автоионизацией воды (Рисунок \ (\ PageIndex {1} \)) и происходит в каждой пробе воды, будь то чистая вода или часть раствора. В какой-то степени автоионизация происходит в любой амфипротической жидкости. (Для сравнения, жидкий аммиак также подвергается автоионизации, но только около 1 молекулы из миллиона миллиардов (1 из 10 15 ) вступает в реакцию с другой молекулой аммиака.)

Рисунок \ (\ PageIndex {1} \) Автоионизация. Небольшая часть молекул воды — примерно 6 из 100 миллионов — самопроизвольно ионизируется в ионы гидроксония и ионы гидроксида.Эта картина обязательно переоценивает степень автоионизации, которая действительно происходит в чистой воде.

Пример \ (\ PageIndex {1} \)

Определить воду как кислоту Бренстеда-Лоури или как основание Бренстеда-Лоури.

  1. H 2 O () + NO 2 (водн.) → HNO 2 (водн.) + OH (водн.)
  2. HC 2 H 3 O 2 (водн.) + H 2 O (ℓ) → H 3 O + (водн.) + C 2 H 3 O 2 (водн.)

Решение

  1. В этой реакции молекула воды отдает протон иону NO 2 , образуя OH (водн.).В качестве донора протонов H 2 O действует как кислота Бренстеда-Лоури.
  2. В этой реакции молекула воды принимает протон от HC 2 H 3 O 2 , превращаясь в H 3 O + (водн.). В качестве акцептора протона H 2 O представляет собой основание Бренстеда-Лоури.

Упражнение \ (\ PageIndex {2} \)

Определить воду как кислоту Бренстеда-Лоури или как основание Бренстеда-Лоури.

  1. HCOOH (водн.) + H 2 O (ℓ) → H 3 O + (водн.) + HCOO (водн.)
  2. H 2 O (ℓ) + PO 4 3- (водн.) → OH (водн.) + HPO 4 2- (водн.)
Ответ

1.H 2 O действует как акцептор протонов (основание Бренстеда-Лоури)

2. H 2 O действует как донор протонов (кислота Бренстеда-Лоури)

Упражнения по обзору концепции

  1. Объясните, как вода может действовать как кислота.
  2. Объясните, как вода может выступать в качестве основы.

Ответы

  1. При правильных условиях H 2 O может отдавать протон, превращая его в кислоту Бренстеда-Лоури.
  2. При правильных условиях H 2 O может принимать протон, что делает его основанием Бренстеда-Лоури.

Ключевые вынос

  • Молекулы воды могут действовать как кислота и основание, в зависимости от условий.

Упражнения

  1. Действует ли H 2 O (ℓ) как кислота или основание?

    H 2 O (ℓ) + NH 4 + (водн.) → H 3 O + (водн.) + NH 3 (водн.)

  2. Действует ли H 2 O (ℓ) как кислота или основание?

    CH 3 (водн.) + H 2 O (ℓ) → CH 4 (водн.) + OH (водн.)

  3. В водных растворах некоторых солей один из ионов соли может реагировать с молекулами воды.В некоторых растворах C 2 H 3 O 2 может протекать следующая реакция:

    C 2 H 3 O 2 (водн.) + H 2 O (ℓ) → HC 2 H 3 O 2 (водн.) + OH ( водн.)

    Действует ли в этой реакции H 2 O в качестве кислоты или основания?

  4. В водных растворах некоторых солей один из ионов соли может реагировать с молекулами воды.В некоторых растворах NH 4 + может протекать следующая реакция:

    NH 4 + (водн.) + H 2 O → NH 3 (водн.) + H 3 O + (водн.)

    Действует ли в этой реакции H 2 O в качестве кислоты или основания?

  5. Почему чистая вода считается нейтральной?

Ответы

  1. основание
  2. кислота
  1. кислота
  2. база

5.Когда вода ионизируется, образуются равные количества H + (кислота) и OH (основание), поэтому раствор не является ни кислотным, ни основным: H 2 O (ℓ) → H + (водный) + OH (водн.)

[БОКОВОЕ ПРИМЕЧАНИЕ: Редко бывает по-настоящему чистая вода. Вода, контактирующая с воздухом, обычно будет слегка кислой, потому что растворенный углекислый газ или угольная кислота снижает pH немного ниже 7. С другой стороны, растворенные минералы, такие как карбонат кальция (известняк), могут сделать воду слегка щелочной.]

кислотно-основная реакция | Определение, примеры, формулы и факты

Кислотно-основная реакция , тип химического процесса, типичным примером которого является обмен одного или нескольких ионов водорода, H + , между частицами, которые могут быть нейтральными (молекулы, такие как вода , H 2 O; или уксусная кислота, CH 3 CO 2 H) или электрически заряженные (ионы, такие как аммоний, NH 4 + ; гидроксид, OH ; или карбонат, CO 3 2-).Он также включает аналогичное поведение молекул и ионов, которые являются кислотными, но не отдают ионы водорода (хлорид алюминия, AlCl 3 и ион серебра AG + ).

сульфат натрия

Сульфат натрия, также называемый глауберовской солью, является, как и другие соли, продуктом кислотно-щелочной реакции.

Martin Walker

Популярные вопросы

Что такое кислоты и основания?

Кислоты — это вещества, содержащие один или несколько атомов водорода, которые в растворе выделяются в виде положительно заряженных ионов водорода.Кислота в водном растворе имеет кислый вкус, меняет цвет голубой лакмусовой бумаги на красный, вступает в реакцию с некоторыми металлами (например, с железом) с выделением водорода, реагирует с основаниями с образованием солей и способствует определенным химическим реакциям (кислотный катализ). Основания — вещества, которые имеют горький вкус и меняют цвет красной лакмусовой бумаги на синий. Основания реагируют с кислотами с образованием солей и способствуют определенным химическим реакциям (щелочной катализ).

Как измеряются кислоты и основания?

Кислотам и основаниям присваивается значение от 0 до 14, значение pH, в зависимости от их относительной силы.Чистая вода, которая является нейтральной, имеет pH 7. Раствор с pH менее 7 считается кислым, а раствор с pH более 7 считается щелочным или щелочным. Сильные кислоты имеют более высокую концентрацию ионов водорода, и им присваиваются значения, близкие к 0. И наоборот, сильные основания имеют более высокие концентрации гидроксид-ионов, и им присваиваются значения, близкие к 14. Более слабые кислоты и основания ближе к значению pH 7, чем их более сильные собратья.

Что происходит во время кислотно-щелочной реакции?

Кислотно-основная реакция — это тип химической реакции, которая включает обмен одним или несколькими ионами водорода, H + , между частицами, которые могут быть нейтральными (молекулы, такие как вода, H 2 O) или электрически заряженные (ионы, такие как аммоний, NH 4 + ; гидроксид, OH ; или карбонат, CO 3 2-).Он также включает аналогичные процессы, которые происходят в молекулах и ионах, которые являются кислотными, но не отдают ионы водорода.

Как кислоты и основания нейтрализуют друг друга (или нейтрализуют)?

Различные реакции дают разные результаты. Реакции между сильными кислотами и сильными основаниями более полно разлагаются на ионы водорода (протоны, положительно заряженные ионы) и анионы (отрицательно заряженные ионы) в воде. Для слабой кислоты и слабого основания нейтрализация более подходящим образом включает прямой перенос протонов от кислоты к основанию.Если один из реагентов присутствует в большом избытке, реакция может дать соль (или ее раствор), которая может быть кислой, основной или нейтральной в зависимости от силы кислот и оснований, реагирующих друг с другом.

Кислоты — это химические соединения, которые в водном растворе проявляют резкий вкус, разъедающее действие на металлы и способность окрашивать некоторые синие растительные красители в красный цвет. Основания — это химические соединения, которые в растворе становятся мыльными на ощупь и окрашивают красные растительные красители в синий цвет. При смешивании кислоты и основания нейтрализуют друг друга и образуют соли, вещества с солоноватым вкусом и без характерных свойств кислот или оснований.

Идея о том, что одни вещества являются кислотами, а другие — основаниями, почти так же стара, как и химия, и термины кислота , основание и соль встречаются очень рано в трудах средневековых алхимиков. Кислоты, вероятно, были первыми из них, которые были распознаны, очевидно, из-за их кислого вкуса. Английское слово acid , французское acide , немецкое Säure и русское kislota происходит от слов, означающих кислый (лат. acidus , немецкий sauer , древнескандинавский sūur и Русский кислый ).К другим свойствам, которые в свое время связывали кислоты, относились их растворяющее или коррозионное действие; их действие на растительные красители; и вспенивание, возникающее при нанесении их на мел (образование пузырьков углекислого газа). Основания (или щелочи) характеризовались в основном своей способностью нейтрализовать кислоты и образовывать соли, причем последние довольно слабо характеризовались как кристаллические вещества, растворимые в воде и имеющие соленый вкус.

Несмотря на их неточный характер, эти идеи служили для корреляции значительного диапазона качественных наблюдений, и многие из самых обычных химических материалов, с которыми сталкивались ранние химики, можно было классифицировать как кислоты (соляная, серная, азотная и угольная кислоты), основания (сода, калий, известь, нашатырный спирт) или соли (поваренная соль, нашатырный спирт, селитра, квасцы, бура).Отсутствие какой-либо очевидной физической основы для рассматриваемого явления затрудняло количественный прогресс в понимании кислотно-основного поведения, но способность фиксированного количества кислоты нейтрализовать фиксированное количество основания была одним из первых примеров химической эквивалентности. : идея о том, что определенная мера одного вещества в некотором химическом смысле равна разному количеству второго вещества. Кроме того, довольно рано было обнаружено, что одну кислоту можно заменить из соли другой кислотой, и это позволило расположить кислоты в приблизительном порядке силы.Также вскоре стало ясно, что многие из этих смещений могут происходить в любом направлении в соответствии с экспериментальными условиями. Это явление свидетельствует о том, что кислотно-основные реакции обратимы, то есть продукты реакции могут взаимодействовать с регенерированием исходного материала. Он также ввел концепцию равновесия в кислотно-щелочную химию: эта концепция гласит, что обратимые химические реакции достигают точки баланса или равновесия, при которой исходные материалы и продукты регенерируются каждой из двух реакций так же быстро, как и они. потребляются другим.

Получите подписку Britannica Premium и получите доступ к эксклюзивному контенту.
Подпишитесь сейчас

Помимо теоретического интереса, кислоты и основания играют большую роль в промышленной химии и в повседневной жизни. Серная кислота и гидроксид натрия входят в число продуктов, производимых в наибольших количествах химической промышленностью, и большой процент химических процессов включает кислоты или основания в качестве реагентов или катализаторов. Почти каждый биологический химический процесс тесно связан с кислотно-щелочным равновесием в клетке или в организме в целом, и кислотность или щелочность почвы и воды имеют большое значение для растений или животных, живущих в них.И идеи, и терминология кислотно-щелочной химии пронизывают повседневную жизнь, и термин соль особенно распространен.

Общие сведения о кислотах и ​​подкислении воды

Что такое pH

Чтобы лучше понять кислоты и их полезное поведение в воде, нам сначала нужно хорошо понимать pH и его значение. Чтобы упростить определение pH, это измерение концентрации ионов водорода [H +] в водном растворе (в этой статье мы будем иметь в виду воду).

Это измерение можно описать следующим образом:

уравнение:

pH = -log [H +] или pH = -log [OH-]

Шкала pH обычно представлена ​​значениями в диапазоне от 0 до 14. Чистая вода имеет значение pH 7, а любой раствор с pH менее 7 (pH от 7 до нуля) считается «кислым», а любой раствор с pH больше 7 (от 7 до 14) считается щелочным или «основным». Шкала pH является логарифмической; на этой шкале каждое изменение целого числа приводит к получению раствора, который в 10 раз более кислый или щелочной.

Примеры:

Значение pH 3 в 10 раз более кислое, чем значение pH 4, а значение pH = 3 в 100 раз (10 раз 10) более кислое, чем pH = 5.

То же самое относится и к растворам алкалаина:

Значение pH 11 в 10 раз более щелочное, чем значение pH 10, а значение pH 11 в 100 раз (10 раз 10) более щелочное значение, чем значение pH 8.

При подкислении воды цель состоит в том, чтобы увеличить концентрацию ионов водорода [H +], чтобы создать их избыток в воде.Это сдвигает равновесие воды, что приводит к уменьшению или понижению pH воды.

Два ключевых аспекта, которые необходимо знать при выборе кислоты

Кислоты можно классифицировать или сгруппировать несколькими различными способами на основе их химических формул, что полезно, когда вы решаете покупать и использовать одну марку кислоты вместо другой. Эти классификации позволяют понять, как кислоты реагируют в воде, и дают представление об их способности фактически подкислять питьевую воду на фермах.

Важно понимать хотя бы одно из двух взаимосвязанных значений и их значение для кислотного воздействия на pH питьевой воды; эти два значения представляют собой 1) K , и затем 2) pKa каждой кислоты.

K a — константа кислотной диссоциации; Проще говоря, K a — это мера силы кислоты в растворе. Чем больше число, тем сильнее кислота. Значения K a часто бывает трудно просто посмотреть и сравнить между кислотами из-за того, что они часто выражаются в научных обозначениях.Здесь в игру вступает значение pKa.

pKa — более простой (и лучший) способ выразить одно и то же значение; взаимосвязь выражается уравнением ниже. При использовании значений pKa , чем отрицательнее число, тем сильнее кислотный .

pKa = -log (K a )

Пример:

Серная кислота (H 2 SO 4 )

K a = 1,0 x 10 -3 или.001

pKa = -log (1,0 x 10 -3 ) = -3

(Использование значений pKa упрощает сравнение различных кислот и их силы.)

Сильные кислоты против слабых

Разница между сильными и слабыми кислотами заключается в их склонности к диссоциации или разделению на протон H + и анион A- в растворе. Сила кислоты количественно определяется ее значением K , и pKa; считается, что сильные кислоты имеют значение K , значение выше 1 и значение pKa ниже 1 (вспомните приведенное выше соотношение).И наоборот, слабые кислоты будут иметь значение K , значение ниже 1 и значение pKa выше 1.

Сильные кислоты полностью диссоциируют в воде, а слабые кислоты частично диссоциируют в воде и остаются в равновесии. Степень этого равновесия зависит от значений K , и pKa кислоты и pH воды. При pH, равном значению pKa слабой кислоты, равновесие будет при соотношении 50% HA (ассоциированного) и 50% H + + A- (диссоциированного).

Сильная кислотная диссоциация:

HA → H + + A

(Кислота полностью диссоциирует на протон и

анион при добавлении в воду.)

Слабая кислотная диссоциация:

HA ↔ H + + A

(Как молекула кислоты [ассоциированная форма], так и протон и анион [диссоциированная форма] находятся в равновесии при добавлении в воду.)

Монпротонная (только одна H + ) кислоты в сравнении с полипротонной (более одной H + ) кислот

Целью добавления кислот в питьевую воду для домашней птицы или свиней является повышение концентрации водородных ионов в воде и, следовательно, снижение pH.Итак, часть молекулы кислоты, которая нас больше всего интересует, — это протон H +. Кислоты снова можно классифицировать с точки зрения того, сколько потенциальных ионов водорода они могут внести в воду; кислота, которая способна отдавать воде единственный ион водорода, называется монопротонной (единичный водород H +). Кислоты, которые могут передавать воде более одного иона водорода, называются полипротонными (два или более свободных протонов водорода). Полипротонные кислоты можно дополнительно классифицировать по тому, сколько атомов водорода они способны отдавать.

Например, серная кислота представляет собой дипротонную (H 2 A — два атома водорода H + ) кислоту; его химическая формула: H 2 SO 4 . Следовательно, он может отдавать до двух ионов водорода.

Теперь, если мы посмотрим на фосфорную кислоту, которая на самом деле является трипротонной (H 3 A — трехводородная) кислотой, она обладает способностью отдавать до трех ионов водорода. Однако то, что кислота состоит из нескольких атомов водорода, не означает, что она всегда будет отдавать все свои ионы водорода воде.Когда в воду добавляются полипротонные кислоты, каждая диссоциация имеет свой собственный набор присвоенных значений K a и соответствующих значений pKa. С каждой последующей диссоциацией эти значения увеличиваются, и диссоциации

становится менее подходящим (или, что труднее использовать больше H + в этой молекуле кислоты). Давайте взглянем на серную и фосфорную кислоты, чтобы лучше понять эту концепцию.

Примеры:

Серная кислота (дипротонная): h3SO4

1-я диссоциация: H 2 SO 4 → H + + HSO 4 — pKA = -3

(сильная кислота)

2-е разъединение: HSO 4 — ↔ H + + SO 4 — pKA = 1.92

(слабая кислота)

Серная кислота — сильная кислота, о чем свидетельствует ее значение pKa, равное -3. Но после первой диссоциации значение pka оставшегося иона гидросульфата больше 1 и, следовательно, фактически является слабой кислотой по своей природе. Следовательно, степень второй диссоциации становится специфически зависимой от pH воды. (Для всех целей, связанных с питьевой водой сельскохозяйственных животных, второе разделение будет почти полным.) Именно эта взаимосвязь делает серную кислоту хорошим выбором для подкисления воды в уже естественных условиях с низким pH.

Фосфорная кислота (трипротонная): h4PO4

Рассмотрение фосфорной кислоты и ее диссоциации в воде (это когда взаимосвязь pKa и pH и то, как она влияет на диссоциацию).

1-е отделение: H 3 PO 4 ↔ H + + H 2 PO 4 — pKa = 2,12

(слабая кислота)

2-е разъединение: H 2 P0 4 — ↔ H + + HPO 4 2- pKa = 7,21

(слабая кислота)

3-е разъединение: HP0 4 2- ↔ H + + PO 4 3- pKa = 12.31

(слабая кислота)

Фосфорная кислота является трипротонной кислотой, поскольку она может отдавать три протона водорода в воду, но степень диссоциации, поскольку это слабая кислота по природе, зависит от pH питьевой воды на ферме, в которую она добавляется. Первое отделение будет почти завершено почти для всей воды на ферме, в которую оно будет добавляться; однако в полевых условиях, когда дело доходит до второй диссоциации, фосфорная кислота становится очень плохим подкислителем из-за взаимосвязи между pH и pka.Значение pKa второй диссоциации для фосфорной кислоты составляет 7,21. Из этого известно, что при pH 7,21 равновесие H 2 PO 4 — и HPO 4 2- будет 50/50. Следовательно, количество отдаваемых протонов водорода ограничено дополнительными 50%, и дальнейшее снижение pH станет очень трудным, если не будут добавлены большие количества дополнительной кислоты для дополнения протонов водорода, образовавшихся при первой диссоциации.

Органические кислоты в сравнении с неорганическими кислотами (минеральные кислоты)

Основное различие между органическими и неорганическими кислотами заключается в наличии углерода в их химическом составе.Органические кислоты (не обязательно) допускаются какой-либо сертификации, такой как OMRI — это обозначение органических кислот строго техническое, а не для сертифицированного органического производства домашней птицы или свиней на убой. Проще говоря, органические кислоты содержат углерод, а неорганические кислоты — нет. Об этом будет важно помнить позже в этой статье. Большинство сильных кислот, используемых в этой области, будут неорганическими кислотами, а большинство смешанных кислотных продуктов будут органическими смесями, которые также будут слабыми кислотами.

Использование кислот на ферме

При использовании кислоты на ферме, независимо от того, используется ли кислота для подкисления воды с целью улучшения качества воды или для увеличения растворимости водорастворимых продуктов (например, антибиотиков), важно знать и понимать, как вода на ферме повлияет на эффективность продуктов. А также то, как добавление этих продуктов в воду может повлиять на состояние водной системы и общую производительность птиц.

Химический состав исходной воды на ферме может существенно повлиять на способность подкислителей снижать pH воды. При добавлении подкислителей в воду на ферме необходимо учитывать щелочность и жесткость воды. Концентрация этих аналитов в воде напрямую влияет на степень сложности или облегчает регулировку pH воды.

Например: два разных участка фермы в непосредственной близости друг от друга могут значительно отличаться (т.е., низкая по сравнению с высокой щелочностью) химический состав воды и продукт, который хорошо работает на одной ферме, могут не принести каких-либо значительных результатов на другой. В общем, чем выше щелочность и жесткость, тем труднее будет изменить pH воды из-за увеличения буферной способности или «нейтрализации» кислоты.

С помощью простых тест-полосок или недорогого датчика pH воды можно легко и быстро провести тестирование на месте с использованием желаемых подкислителей, чтобы измерить количество продукта, которое необходимо добавить для достижения желаемого диапазона pH.Для достижения наилучших результатов при выборе торговой марки / продукта следует учитывать как химический состав воды местных сельскохозяйственных скважин, так и свойства подкислителя.

Заключительные замечания

Каждый раз, когда продукт добавляется в воду, которая в конечном итоге будет потреблена птицами, очень важно понимать, что добавляется и как это может повлиять на систему водоснабжения и продуктивность птицы. Ранее в этой статье объяснялась разница между органическими и неорганическими кислотами, и эта разница заключается в наличии углерода в химическом составе кислоты.На первый взгляд эта разница не кажется такой уж значительной; однако углерод может использоваться в качестве источника пищи для любой микробиологии, которая может жить внутри всей водной системы (например, регуляторов давления, напорных резервуаров, стояков, линий поения). Этот потенциальный источник пищи, наряду с изменением pH, может создать идеальную среду для «биологического цветения» внутри ватерлинии, эффективно перекрывая ватерлинии и перекрывая подачу воды птицам. Некоторые, читающие эту статью, возможно, уже испытали это на своих собственных фермах.

При использовании органических кислот рекомендуется постоянно дезинфицировать воду или следить за использованием органических кислот с биоцидными молекулами, такими как хлорноватистая кислота (таблетка), активированный кислотой диоксид хлора, биоцидный пероксид, зарегистрированный EPA, или EPA. -регистрация перуксусной кислоты, чтобы предотвратить появление этого цветения. Использование дезинфицирующих средств для водопровода между каждым стадом имеет решающее значение при использовании органических кислот / смесей для простого регулирования pH или обеспечения оптимального закисления желудочно-кишечного тракта для здоровья кишечника птицы.

Последнее, что нужно учитывать, — это состав самой кислоты, а также учесть образующиеся анионы [A], оставшиеся в воде, и последующее воздействие, которое они могут оказать на птиц. Одним из примеров может быть использование продукта на основе серной кислоты и большого количества кислоты, которое может потребоваться для снижения pH до желаемого диапазона в сильно забуференной воде. Этот подход редко, но может привести к значительному увеличению количества сульфат-ионов, присутствующих в воде. Высокий уровень сульфатов и взаимодействие с магнием в воде может оказывать на птиц слабительное действие, отрицательно влияя на продуктивность.Это всего лишь один пример из многих потенциальных ловушек и взаимодействий, которые необходимо учитывать при добавлении подкислителей или любого продукта в воду на ферме.

Очистить воду несложно, но вам нужно сначала узнать и понять химический состав питьевой воды на вашей ферме, а затем обратиться за хорошим советом к авторитетному поставщику технических услуг, который действительно поможет (а не просто продаст вам «универсальный вариант»). все, подход «один продукт-все-все»). Если у вас есть вопросы, связанные с кислотами и подкислением воды, обратитесь к своему менеджеру по охране здоровья животных MWI.Мы будем рады помочь!

Теплица и цветоводство: Регулировка щелочности с помощью кислот

Кислоты были и всегда будут отличным инструментом для растениеводов, позволяющим лучше контролировать щелочность поливной воды (в основном бикарбонаты и карбонаты) и pH среды выращивания. Как только станет понятна роль щелочности, производитель может рассмотреть следующий практический шаг по контролю щелочности с помощью кислот через систему инжектора.

Кислота Тип

Кислоты, обычно доступные производителям, включают фосфорную, серную, азотную и лимонную.В таблице 1 перечислены критерии выбора кислоты, подходящей для вашей ситуации: относительная безопасность, нейтрализующая способность, стоимость и содержание питательных веществ. По нашему опыту, наиболее эффективной и широко используемой кислотой является серная кислота; однако это одна из самых опасных кислот для использования. Для удаления небольшого количества щелочности фосфорная кислота может быть предпочтительной кислотой. Однако мы не рекомендуем добавлять более 2,25 жидких унций этой кислоты на 100 галлонов воды из-за количества фосфора, которое можно добавить. Азотная кислота теоретически идеальна, потому что она добавляет нитратный азот; но он дымится и сильно окисляет, что затрудняет обращение с ним.Лимонная кислота — это слабая органическая кислота и твердое вещество, что делает ее более безопасной, чем три других; но он гораздо менее эффективен и, следовательно, дороже в использовании.

Используйте правильный инжектор

После того, как вы выберете кислоту для использования, убедитесь, что ваш инжектор справится с этой задачей. Прочтите руководство по эксплуатации форсунки, чтобы получить эту информацию, или позвоните производителю форсунки. Примечание. Некоторые производители инжекторов заявляют, что можно использовать максимум 5% кислоты. Это соответствует примерно 6 жидким унциям кислоты на галлон воды — необычно высокая концентрация кислоты.

Расчет количества кислоты для использования

Мы рекомендуем использовать достаточно кислоты, чтобы снизить щелочность воды до целевого диапазона. В таблице 2 представлены рекомендуемые целевые диапазоны щелочности в зависимости от размера контейнера. Во-первых, сделайте анализ воды на щелочность. Вы можете провести лабораторный тест на щелочность или использовать набор, чтобы измерить ее самостоятельно (наборы для проверки щелочности можно приобрести у тепличных или научных дистрибьюторов). Затем рассчитайте количество кислоты, необходимое для доведения воды до целевого диапазона щелочности.(Текущая щелочность — желаемая щелочность = щелочность, которую необходимо нейтрализовать). В таблице 1 указано количество кислоты, которое следует использовать для определенной щелочности ppm на 100 галлонов воды. Теперь вы готовы к пилотному калибровочному запуску.

Таблица 1. Характеристики кислот
, применяемых для нейтрализации щелочности воды.
Тип кислоты Типичная прочность Относительная опасность Содержание питательных веществ (ppm) z Нейтрализующая сила Удельный вес мл кислоты / ppm щелочности / 100 галлонов x
Фосфор 75% w Умеренный 25.6 P, as PO 4 45,0 u 1,381 0,70
серная 93% против Высокая 43,6 S, как SO 4 136,0 1,835 0,23
азот 63% Высокая 14,6 N, as NO 3 52,3 1,381 0,56
лимонный 100% Низкий Нет НЕТ НЕТ НЕТ

z Содержание питательных веществ в 1 жид.унция. добавляется в 100 галлонов воды. Внесите соответствующие изменения в программу внесения удобрений.

y Количество нейтрализованной щелочности (мг CaCO3 / литр) при 1 жид. унция. кислоты добавляется на 100 галлонов воды.

x Коэффициент преобразования силы кислоты при указанном удельном весе. Пример: если у вас щелочность 250 и вы хотите достичь целевого значения 150, вам необходимо нейтрализовать 100 мг CaCO 3 / литр. Если использовать серную кислоту, то 100 х 0.23 = 23 миллилитра (мл) / 100 галлонов. Необходимо 23 мл / 29,6 мл / фл. унция. = 0,77 (0,75 жидких унций) / 100 галлонов. Нормы будут зависеть от точной прочности и удельного веса.

w Фосфорная кислота бывает разной концентрации, но наиболее распространена 75%. По возможности используйте тяжелый свободный или пищевой сорт.

v 93% серная кислота также известна как 66 be ‘(Baume’) кислота. Некоторые рекомендуют аккумуляторный кислотный электролит, его концентрация составляет около 35%.

u Предполагается, что около одной трети кислоты эффективна, поскольку фосфорная кислота не полностью диссоциирует.

Безопасность прежде всего

Кислоты — опасные химические вещества. Когда концентрированные кислоты смешиваются с водой, выделяется огромное количество тепла (которое может даже деформировать или расплавить пластик). Неправильное смешивание может привести к травмам. При использовании кислот всегда надевайте соответствующее защитное снаряжение. Сюда входят защитные очки, маска для лица, прорезиненный фартук или комбинезон, а также кислотостойкие перчатки и обувь. Вы должны быть в состоянии найти дистрибьюторов защитного оборудования в Желтых страницах в разделе «Безопасность».При хранении, смешивании и обращении с кислотами необходимо соблюдать федеральные и государственные законы и нормы техники безопасности.

Правильное смешивание

Используйте кислотоупорные контейнеры для хранения кислотного основного раствора. Подойдут сверхмощные полиэтиленовые мусорные баки.

Всегда смешивайте кислоту с водой. Заполните емкость для материала примерно наполовину от конечного объема, который вы хотите смешать с водой. (Примечание: поскольку это пробный запуск, вам не нужно готовить полное количество подкисленного исходного раствора, потому что вы, возможно, захотите отрегулировать количество кислоты или добавить удобрение в основной раствор позже.) Тщательно отмерьте кислоту, используя хорошую мерную емкость. Затем медленно и осторожно добавьте кислоту в воду до центра поверхности воды. При дозировании кислоты из большой бочки или контейнера вам следует приобрести кислотостойкое насосно-дозирующее устройство с ручным приводом («Промышленные поставщики в Желтых страницах»). Во время и после добавления кислоты в воду необходимо размешать кислоту в воде . Кислота тяжелее воды, поэтому не думайте, что она легко смешается только потому, что это жидкость.Размешивать! Избегайте разбрызгивания!

Таблица 2. Рекомендуемые нормативы щелочности (мг CaCO 3 / литр) z .
Размер контейнера Допустимая щелочность Уровень концерна y
Заглушки 60-100 <40,> <40,> 120
Горшки 80-120 <40,> 140
Горшки 4-5 дюймов 100-140 <40,> 160
горшки> 6 дюймов 120-180 <60,> 200

z Уровни щелочности, рекомендованные Scotts Testing Lab.Фактические уровни могут варьироваться в зависимости от типа культуры и желаемой реакции растений.

y Низкие уровни могут привести к снижению pH среды, а высокие уровни могут привести к увеличению pH среды. Эти тенденции сильно зависят от нормы удобрения.

Пилотный прогон или измерение «Калибровка»

После приготовления подкисленного маточного раствора следует определить, достигли ли вы целевой щелочности поливной воды для вашего применения.Запустите инжектор при соответствующем коэффициенте разбавления на 5–10 минут, затем возьмите образец. Лучше всего налить воду, которую вы хотите протестировать, в 5-галлонное ведро и взять образец из 5-галлонного ведра. Проверьте щелочность. При необходимости внесите изменения. Как только вы закончите, разумно отправить еще один образец подкисленной воды в аналитическую лабораторию для проведения полного теста. Это проинформирует вас, изменилось ли что-нибудь еще, кроме щелочности.

Теперь удобрения

Многие производители хотят использовать один инжектор и смешивать кислоту с удобрениями.Использование фосфорной, азотной и лимонной кислот совместимо с влажными водорастворимыми удобрениями. Серная кислота несовместима с кальцийсодержащими удобрениями, такими как нитрат кальция, или такими препаратами, как 15-0-15 и 17-0-17 в концентрированной форме.

Смешивание удобрений с кислотой

Если вы разбавляете кислоту из отдельного инжектора, пропустите этот шаг. Помните, что вы добавляете только часть кислоты для выполнения калибровочного цикла (половина объема основного раствора). Добавьте остаток кислоты до общего количества кислоты, которое вы хотите получить.Вы можете добавить больше воды, оставив «место» для добавления удобрений. Добавляйте удобрение осторожно, чтобы избежать разбрызгивания, и достаточно воды, чтобы достичь конечного объема — тщательно перемешайте. Опять же, проверьте закачку подкисленного питательного раствора, чтобы убедиться, что поливная вода находится в пределах целевого диапазона щелочности. Готово!

a Оригинальная статья, автором которой являются Рик Ветановец и Шеннен Ферри, The Scotts Company, Мэрисвилл, Огайо

Ресурсы

AlkCalc — Этот калькулятор дает рекомендации по количеству кислоты, которую следует добавить в поливную воду, чтобы изменить уровни pH и щелочности.Он также обеспечивает количество добавленных фосфора, азота и серы, которое будут обеспечивать соответствующие кислоты, а также экономическое сравнение каждой кислоты.

Вода, кислоты и основания

Вода,
Кислоты и основания


Кислотно-основная химия
воды

В химии водных растворов преобладают
равновесие между нейтральными молекулами воды и ионами, которые они
форма.

2 H 2 O ( л )
H 3 O + ( водн. ) + OH ( водн. )

Строгое соблюдение правил записи равновесия
постоянное выражение этой реакции дает следующие
результат.

Это законное выражение константы равновесия, но оно
не учитывает огромную разницу между
концентрации нейтральных молекул H 2 O и H 3 O +
и OH ионов в равновесии.

Измерения способности воды проводить электрический ток.
В настоящее время предполагается, что чистая вода при температуре 25 o C содержит 1,0 x
10 -7 моль на литр каждого из этих ионов.

[H 3 O + ] =
[OH ] = 1,0 x 10 -7 M

При той же температуре концентрация нейтрального H 2 O
молекул составляет 55,35 моль.

Соотношение концентраций H + (или OH )
иона до концентрации нейтральных молекул H 2 O
следовательно, 1,8 x 10 -9 .

Другими словами, только около 2 частей на миллиард (ppb)
молекулы воды диссоциируют на ионы при комнатной температуре.

Равновесная концентрация молекул H 2 O равна
намного больше, чем концентрации H 3 O +
и ионов OH , что эффективно постоянно. Мы
поэтому поместите член [H 2 O] в равновесие
постоянна для реакции и тем самым значительно упрощает
расчет равновесия. Начнем с перестановки равновесия
постоянное выражение для диссоциации воды, чтобы дать
следующее уравнение.

[H 3 O + ] [OH ] = K c
x [H 2 O] 2

Затем заменим член в правой части этого уравнения
с константой, известной как водная диссоциация
константа равновесия
, K w .

[H 3 O + ] [OH ] = K w

В чистой воде при 25 ° C [H 3 O + ] и
Концентрация ионов [OH ] равна 1.0 x 10 -7 M .
Таким образом, значение K w при 25 ° C составляет
1,0 x 10 -14 .

[1,0 x 10 -7 ] [1,0 x 10 -7 ] = 1,0 x 10 -14 (при 25 ° C)

Хотя K w определяется в терминах
диссоциации воды, это выражение константы равновесия
одинаково справедливо для растворов кислот и оснований, растворенных в
вода.Независимо от источника H 3 O +
и OH ионов в воде, продукт
концентрации этих ионов в равновесии при 25 ° C всегда
1,0 x 10 -14 .


Сильные кислоты и
H
3 O +
и ОН

Концентрации ионов

Предположим, мы добавляем достаточно сильной кислоты в стакан с водой, чтобы
увеличить концентрацию ионов H 3 O + до 0.010 М .
Согласно принципу Ле-Шателье, это должно стимулировать
равновесие между водой и ее ионами слева, уменьшая
количество ионов H 3 O + и OH ионов в
решение.

2 H 2 O ( л )
H 3 O + ( водн. ) + OH ( водн. )

Потому что так много ионов H 3 O + в
в этом растворе изменение концентрации этого иона слишком велико.
маленький, чтобы заметить.Когда система возвращается в состояние равновесия, H 3 O +
концентрация ионов по-прежнему составляет около 0,010 M . Более того,
когда реакция возвращается к равновесию, продукт H 3 O +
и концентрация ионов OH снова равна K w .

[H 3 O + ] [OH ] =
1 х 10 -14

Таким образом, раствор возвращается к равновесию, когда
диссоциация воды настолько мала, что ион OH
концентрация всего 1.0 x 10 -12 M .

Таким образом, добавление кислоты в воду влияет на
концентрация как H 3 O + , так и OH
ионы. Поскольку это источник этого иона, добавление кислоты в воду
увеличивает концентрацию H 3 O +
ион. Однако добавление кислоты к воде снижает степень
вода диссоциирует. Следовательно, это приводит к значительному
снижение концентрации иона ОН .

Как и следовало ожидать, обратный эффект наблюдается, когда
основание добавляется в воду. Поскольку мы добавляем базу, OH
концентрация ионов увеличивается. Как только система вернется в
равновесие, продукт H 3 O + и OH
концентрация ионов снова равна K w .
Разумеется, единственный способ добиться этого — уменьшить
концентрация иона H 3 O + .

.

Добавить комментарий

Ваш адрес email не будет опубликован. Обязательные поля помечены *